Sie sind auf Seite 1von 64

Situation: During an Anatomy and Physiology class, the lecturer discussed about the female reproductive system. 1.

Estrogen, one of the hormones regulating cyclic activities in female reproductive system is responsible for which effect? a. Increases the quantity and pH of cervical mucus, causing it to become thin and watery and can be stretched to a distance of 10-13 cm. b. Inhibits the production of LH c. Increases endometrial tortuosity d. All of the above * Effects of estrogen: Inhibits the production of FSH Causes hypertrophy of the myometrium Increases the quantity and pH of cervical mucus, causing it to become thin and watery and can be stretched to a distance of 10-13 cm. Effects of Progesterone Inhibits the production of LH Increases endometrial tortuosity Increased endometrial secretions Facilitates transport of the fertilized ovum through the fallopian tubes 2. Jessa, 17 years old, is bleeding between periods of less than two weeks. This condition is an abnormality in the menstrual cycle known as: a. Metrorrhagia b. Menorrhagia c. Amenorrhea d. Dysmenorrheal * Abnormalities of Menstruation 1. Amenorrhea absence of menstrual flow 2. Dysmenorrhea painful menstruation 3. Oligomenorrhea scanty menstruation 4. Menorrhagia -excessive menstrual bleeding 5. Metrorrhagia bleeding between periods of less than 2 weeks 3. One factor of having a normal delivery is the size of the pelvis. Pelvis serves as the passageway for the passenger (fetus) during childbirth. The most ideal pelvis for childbirth is: a. Android b. Anthropoid c. Platypelloid d. Gynecoid * Gynecoid is the normal female pelvis. The inlet is well rounded. This is the most ideal pelvis for childbirth. Android male pelvis. Inlet has a narrow, shallow posterior portion and pointed anterior

portion. Anthropoid transverse diameter is narrow and anteroposterior (AP) diameter of this pelvis is larger than normal. Platypelloid inlet is oval while AP diameter of this pelvis is shallow. 4. An important landmark of the pelvis that determines the distance of the descent of the head is known as: a. Linea terminalis b. Sacrum c. Ischial spines d. Ischial tuberosities * Ischial spines are the point of reference in determining the station (relationship of the fetal presenting part to the ischial spines). When the fetal head is at the level of the ischial spines the station is zero. When it is 1 cm above the ischial spines it is -1 and if 1 cm below the ischial spines it is +1. 5. a. b. c. d. The permanent cessation of menstruation is: Amenorrhea Menopause Oligomenorrhea Hypomenorrhea

* The keyword here is permanent cessation. Thus, menopause is the correct answer. Amenorrhea is a temporary cessation of menses. Oligomenorrhea is a menstruation with scanty blood flow. Hypomenorrhea is an abnormally short duration of menstruation. Situation: Mrs. Donna, pregnant for 16 weeks age of gestation (AOG), visits the health care facility for her prenatal check-up with her only son, Mark. During assessment the client told the nurse that previously she got pregnant twice. The first was with her only child, Mark, who was delivered at 35 weeks AOG and the other pregnancy was terminated at about 20 weeks AOG. 6. Based on the data obtained, Mrs. Donnas GTPAL score is: a. 20111 b. 21111 c. 30111 d. 31111 * Gravida (G) number of pregnancy Term (T) number of full-term infants born (born at 37 weeks or after) Para (P) number of preterm infants born (born before 37 weeks) Abortion (A) number of spontaneous or induced abortions (pregnancy terminated before the age of viability). Age of viability is 24 weeks. Living children (L) number of living children. (Source: Maternal and Child Health Nursing by Adelle Pillitteri, 5th Ed. P.252)

Since Mrs Donna has two previous pregnancies and is presently pregnant (16 weeks), G is 3. Mark, her only child was born at 35 weeks AOG which falls under the preterm category. Thus, T is zero and P is 1. The other pregnancy was terminated at 20 weeks AOG which falls under abortion, hence A=1. Mark is her only living child, thereby, L=1. Her GTPAL score is: 30111, G=3 T=0 P=1 A=1 L=1 7. Discomforts during pregnancy are discussed by the nurse to the Mrs. Donna. Which of the following, when complained by the client would alert the nurse? a. Easy fatigability b. Nausea and vomiting c. Edema of the lower extremities d. Heartburn * 8. Psychological and emotional responses of pregnant women differ. However, general emotional response has been noted during pregnancy based on their gestational age. Mrs. Donna will most likely have which emotional response towards her pregnancy? a. Presents denial disbelief and sometimes repression. b. Has personal identification of the baby and realistic plans for future of the child. c. Fantasizes the appearance of the baby. d. Verbalizes fear of death during childbirth. 9. The nurse assisted Mrs. Donna to a dorsal recumbent position and is about to assess the fetal heart rate (FHR). Which of the following apparatus should the nurse use in auscultating for the FHR? a. Doppler apparatus b. Fetoscope c. Ultrasound d. Stethoscope Morning sickness characterized by nausea and vomiting is only noted during the FIRST TRIMESTER of pregnancy (first 3 months). Excessive nausea and vomiting which persists more than 3 months is a condition called Hyperemesis gravidarum that requires immediate intervention to prevent starvation and dehydration. Management for hyperemesis gravidarum includes the administration of D5NSS 3L in 24 hours and complete bed rest. Easy fatigability is a consequence of the physiologic anemia of pregnancy (physiologic meaning it is normally expected during pregnancy, thus A is incorrect). Edema of the upper extremities not the lower extremities should alert the nurse because of the possibility of toxemia, hence C is incorrect. Heartburn during pregnancy is due to the increase progesterone which decreases gastric motility causing a reversed peristaltic wave leading to regurgitation of the stomach contents through the cardiac sphincter into the esophagus, causing irritation.

10. Mrs. Donna asked the nurse, when a fetal heart starts beating. The nurse correctly responded by stating: a. 3 weeks AOG b. 8 weeks AOG c. 12 weeks AOG d. 20 weeks AOG * Fetal heart starts beating at 3 weeks AOG. The heart at this time is consisting of two parallel tubes. By 8 weeks AOG, fetal heartbeat can be detected with an ultrasound. During 12 weeks AOG, the fetal heart rate is audible with a Doppler apparatus. A fetal heart beat is detectable with fetoscope by the 20th week AOG. (Source: Foundations of Maternal-Newborn Nursing by Murray and McKinney/Saunders 4th Ed.) Situation: Mrs. Dela Cruz is in labor and is brought to the emergency room with a ruptured bag of water. 11. a. b. c. d. The nurses initial action once the bag of water has ruptured is: Take the fetal heart tones Put the client to the bed immediately Perform an IE Take the womans temperature

* The keyword is INITIAL ACTION. The important consideration before answering the question is to take a look at the situation. SITUATION: THE WOMAN IS IN THE Emergency Room or is seeking admission. A woman in labor seeking admission to the hospital (in the ER) and saying that her BOW has ruptured should BE PUT TO BED IMMEDIATELY and the fetal heart tones taken consequently. If a woman in the Labor Room says that her membranes have ruptured, the initial nursing action is to take the fetal heart tone. 12. Mrs. Dela Cruzs has contractions growing stronger which lasts for 40-60 seconds and occur approximately every 3-5 minutes. The doctor is about to perform an IE, the nurse expects that the clients cervical dilatation will be: a. 0-3 cm b. 4-7 cm c. 8-10 cm d. 11-13 cm * The nurse would expect that the clients cervical dilatation is 4-7 cm as the contraction duration and interval is noted for clients who are in the active phase of the first stage of labor. The maximum cervical dilatation is 10 cm, thus, letter D should be eliminated first. The first stage of labor (stage of dilatation) is divided into three phases. Latent phase 0-3 cm cervical dilatation; contractions are short and mild lasting 20-40 seconds and occurring approximately every 5-10 minutes. Active phase 4-7 cm cervical dilatation; contractions grow stronger, lasting 40-60 seconds and occur at approximately every 3-5 minutes.

Transition phase 8-10 cm cervical dilatation; contractions reach their peak of intensity, occurring every 2-3 minutes with a duration of 60-90 seconds. 13. The doctor informed the woman that she is on station -1. Mrs. Dela Cruz asked the nurse, what does a station -1 means, the most appropriate response of the nurse is: a. It means that engagement has already occurred. b. The presenting part of your baby is at the entrance of the true pelvis or the largest diameter of the presenting part into the true pelvis. c. Your baby is still floating or ballotable d. The presenting part of your baby is at the vulvar ring of your reproductive organ. *. Station -1 means that the fetal presenting part is above the level of the ischial spines. Letter A is wrong because engagement is described as Station 0. Letter B is incorrect because the statement of nurse is describing the occurrence of engagement that is again station 0. Prior to engagement the fetus is said to be "floating" or ballottable, thus letter C is the best option. Letter D, is describing crowning which is described as Station +3 or +4. 14. The history of Mrs. Dela Cruz revealed that she is a multipara. When should the nurse transport the client from the labor room to the delivery room? a. When the cervical dilatation is 8 cm. b. When the cervical dilatation is 10 cm. c. When the cervical dilatation is 9 cm. d. When the client feels the urge to push. * Multiparas are transported to the DR when the cervical dilatation is 7-8 cm because in multiparas dilatation may proceed before effacement is completed. Effacement must occur at the end of dilatation, however, before the fetus can be safely pushed through the cervical canal; otherwise, cervical tearing could result. Primiparas are transported to the DR when the cervical dilatation is 9-10 cm. 15. Monitoring contractions is very important during labor. To monitor uterine contractions, what should the nurse do? a. Observe for the clients facial expression to know that the contraction has started or stopped. b. Instruct the client take note of the duration of her contractions. c. Offer ice chips to the woman. d. Spread the fingers lightly over the fundus to monitor the contraction. ** The nurse should spread his/her fingers lightly over the fundus to monitor the uterine contractions 16. Uterine contractions can occur because of the interplay of the contractile enzyme adenosine triphosphate and the influence some hormones. Which of the following least likely contributes to the occurrence of uterine contractions? a. Oxytocin b. Estrogen

c. Prolactin d. All of the above. *Prolactin is the hormone that produces milk in mammary glands. Uterine contractions can occur because of the interplay of the contractile enzyme adenosine triphosphate and the influence some hormones and major electrolytes which are the following: Calcium Sodium Potassium Specific contractile proteins (actin and myosin) Epinephrine and norepinephrine Oxytocin Estrogen and progesterone Prostaglandins 17. a. b. c. d. Dysfunctional labor may be caused by which of the following? Excessive or too early analgesia administration Exhausted mother Overdistention of the uterus All of the above

*Dysfunctional Labor is caused by the ff: Inappropriate use of analgesia Pelvic bone contraction that has narrowed the pelvic diameter so that a client cant pass (e.g. in a client with rickets) Poor fetal position Extension rather then extension of the fetal head Overdistention of the uterus Cervical rigidity Presence of a full rectum or bladder Mother becoming exhausted from labor Primigravid status 18. The clients uterine contractions are hypotonic. The nurses top priority with hypotonic contractions during the intrapartal period is: a. Pain relief b. Psychological support c. Monitoring the lochia for possible bleeding d. Infection control *When the contractions are hypotonic, the length of labor is increased. When the cervix is dilated for a long period of time, both the uterus and fetus are at greater risk of infection. Hypotonic contractions are not exceedingly painful because of their lack of intensity. Monitoring of bleeding through evaluation of lochia is done during the postpartum period not the intrapartum period.

19. a. b. c. d.

For a woman experiencing hypotonic contractions, what should be done initially? Obtain an ultrasonic result Infusion of oxytocin Administration of analgesia Amniotomy

* Initially, the nurse should obtain an ultrasonic confirmation ruling out a CPD or cephalopelvic disproportion. Thus, A is the best answer. Oxytocin is infused after the CPD is ruled out, because if CPD is present CS will be done. Analgesic administration will further decrease the intensity of uterine contractions as its inappropriate use is one of the reasons why hypotonic contractions occur. Amniotomy (artificial rupture of membrane) may be done after oxytocin is infused to speed up the labor 20. The most important nursing consideration in a postpartal woman with a hypotonic contraction is: a. Assessment for infection b. Assessment for bleeding c. Assessment for FHR d. Assessment for womans coping mechanism * During the postpartum period, the uterus should be palpated and lochia should be assessed because contractions after birth may also be hypotonic that will result to bleeding. Situation: Bleeding during pregnancy is a serious case and should be managed immediately. 21. Mrs. Diane is diagnosed with Placenta Previa. The main difference with the bleeding in placenta previa and abruption placenta is that placenta previa has: a. Painful bleeding b. Rigid abdomen c. Bright-red blood d. Blood filled with clots * In placenta previa the bleeding that occurs is abrupt, painless, bright-red and sudden to frighten a woman. With abruption placenta, the bleeding is painful, the abdomen is rigid or board-like and the blood is dark-red or filled with clots. 22. In caring for a client diagnosed with placenta previa, the nurse should avoid which of the following? a. Inspecting the perineum b. Performing a Kleihauer-Betke test c. Performing a pelvic examination d. All of the above * Never attempt a pelvic or rectal examination with painless bleeding late in pregnancy because any agitation of the cervix when there is placenta previa may initiate massive hemorrhage,

possibly fatal to both the mother and the fetus. The perineum should be assessed or observed or inspected for bleeding by looking over the

23. For the nurse to distinguish that the bleeding of the patient is placenta previa or abruption placenta what should she ask the woman? a. Whether there was accompanying pain b. What she has done for bleeding c. Estimation of blood loss d. All of the above * placenta previa presents bleeding without pain whilst the bleeding in abruptio placenta is painful. 24. Continued bleeding may result to fetal distress. The nurse knows that the fetus is being compromised when she observed or note which of the following: a. Fetal tachycardia b. Fetal bradycardia c. Fetal thrashing d. All of the above * Signs of fetal distress include: tachycardia, bradycardia, fetal thrashing and meconium-stained amniotic fluid. 25. A woman in labor is diagnosed with abruption placenta. The nurse would expect which findings in the clients history that may contribute to the occurrence of the complication? a. Age of 24 years old b. Cigarette smoking c. Sleeping 8 hours per night d. Sitting for long period * Predisposing factors for abruptio placenta: Advanced maternal age Short-umbilical cord Chronic hypertensive disease PIH Direct trauma Vasoconstriction from cocaine or cigarette use

SITUATION : [ND89] Aling Julia, a 32 year old fish vendor from baranggay matahimik came to see you at the prenatal clinic. She brought with her all her three children. Maye, 1 year 6 months; Joy, 3 and Dan, 7 years old. She mentioned that she stopped taking oral contraceptives several months ago and now suspects she is pregnant. She cannot remember her LMP. 1. Which of the following would be useful in calculating Aling Julia's EDC? [3] A. Appearance of linea negra B. First FHT by fetoscope C. Increase pulse rate D. Presence of edema * The answer of some people is A because they say linea negra appears at 2nd trimester. Appearance of Linea negra is not the same with all women. Some will have it as early as first trimester while other on the 2nd trimester. It is very subjective and non normative. However, First FHT by fetoscope is UNIVERSAL and it is arbitrarily accepted that it starts at the 4th month of gestation. Therefore, If I heard the First FHT by fetoscope, I can say that Aling Julia's EDC is at 4th month and the EDC will be around 5 months from now.Pulse rate and Edema will never suggest the estimated date of confinement nor age of gestation. 2. Which hormone is necessary for a positive pregnancy test? [1] A. Progesterone B. HCG C. Estrogen D. Placental Lactogen * HCG is responsible for positive pregnancy test. But it is NOT a positive sign of pregnancy. Only PROBABLE. Purpose of HCG is to maintain the secretion of progestrone by the corpus luteum. It will deteriorate by 2nd trimester as the placenta resumes its funciton. HCG is also use to stimulate descend of the testes in case of cryptorchidism or undescended testes. HCG peaks at 10 weeks then decline for the rest of the pregnancy. Non pregnant females will have less than 5 mIU/ml and can reach up to 100,000 mIU/ml in pregnant women. By the way, undescended testes repair is done when the child is 1 year old according to Lippinncots, the doctor will try to wait baka kasi bumaba pa before they do surgery. 3. With this pregnancy, Aling Julia is a [1] A. P3 G3 B. Primigravida C. P3 G4 D. P0 G3

* She has 3 children, so para 3. Since she is pregnant, this is her 4th gravida. Remember that even if the pregancy is beyond the age of viability [ >7 months ] consider it as PARA and not GRAVIDA as long as the baby is still inside the uterus. A common error of the old nurses in a puericulture center where I dutied in is that they count the child inside the mother's womb as GRAVIDA when it is greater than 7 months! [ kawawang nanay, mali na ang home based mothers record mo ] I tried to correct it but they still INSISTED. I read pillitteri thinking that I might be wrong nakakahiya naman... but I was right. 4. In explaining the development of her baby, you identified in chronological order of growth of the fetus as it occurs in pregnancy as [1] A. Ovum, embryo, zygote, fetus, infant B. Zygote, ovum, embryo, fetus, infant C. Ovum, zygote, embryo, fetus, infant D. Zygote, ovum, fetus, embryo, infant * The Ovum is the egg cell from the mother, the sperm will fertilize it to form a zygote. This usually happens in the AMPULLA or the distal third of the fallopian tube. Hyalorunidase is secreted by the sperm to dissolve the outer memberane of the ovum. The zygote now containes 46 chromosomes. 23 from each germ cell. The zygote is now termed as an embryo once it has been implanted. Implantation takes 3-4 days. When the embryo reach 8th weeks, it is now termed as a FETUS until it has been delivered and then, neonate then infant. 5. Aling Julia states she is happy to be pregnant. Which behavior is elicited by her during your assessment that would lead you to think she is stressed? [3] A. She told you about her drunk husband B. She states she has very meager income from selling C. She laughs at every advise you give even when its not funny D. She has difficulty following instructions * Stressed is manifested in different ways and one of them, is difficulty following instructions. Telling you that her husband is drunk and has meager income from selling is not enough for you to conclude she is stressed. Assessment is always based on factual and specific manifestations. A diagnosis is made from either ACTUAL or POTENTIAL/RISK problems. A and B are both potential problems, but not actual like D. C is automatically eliminated first because laughing is not indicative of stress. 6. When teaching Aling Julia about her pregnancy, you should include personal common discomforts. Which of the following is an indication for prompt professional supervision? [2] A. Constipation and hemorrhoids B. Backache C. Facial edema D. frequent urination

*Facial edema is NOT NORMAL. Facial edema is one sign of MILD PRE ECLAMPSIA and prompt professional supervision is needed to lower down the client's blood pressure. Blood pressure in Mild Pre Eclampsia is around 140/90 and 160/110 in severe. Treatment involves bed rest, Magnesium sulfate, Hydralazine, Diazoxide and Diazepam [ usually a combination of Magsul + Apresoline [ Hydralazine ] ] Calcium gluconate is always at the client's bed side when magnesium toxicity occurs. It works by exchanging Calcium ions for magnesium ions. A,B and D are all physiologic change in pregnancy that do not need prompt professional supervision. Frequent urination will disappear as soon as the pressure of the uterus is released against the bladder and as soon as the client's blood volume has returned to normal. Backache is a common complaint of women with an OCCIPUT POSTERIOR presentation due to pressure on the back. Intervention includes pelvic rocking or running a tennis ball at the client's back. Constipation and hemorrhoids are relieved by increasing fluid intake and hot sitz bath. 7. Which of the following statements would be appropriate for you to include in Aling Julia's prenatal teaching plan? [1] A. Exercise is very tiresome, it should be avoided B. Limit your food intake C. Smoking has no harmful effect on the growth and development of fetus D. Avoid unnecessary fatigue, rest periods should be included in you schedule * Exercise is not avoided in pregnancy, therfore eliminate A. Food is never limited in pregnancy. Calories are even increased by around 300 cal a day as well as vitamins and minerals. Smoking, alcohol and drug use are avoided for the rest of the pregnancy because of their harmful effects on the growth and development of the fetus. Rest period and avoiding unecessary fatigue is one of the pillars in health teaching of the pregnant client. 8. The best advise you can give to Aling Julia regarding prevention of varicosities is [3] A. Raise the legs while in upright position and put it against the wall several times a day B. Lay flat for most hours of the day C. Use garters with nylon stocking D. Wear support hose * A thigh high stocking or a support hose WORN BEFORE GETTING UP in the morning is effective in prevention of varicosities. Stocking should have NO GARTERS because it impedes blood flow, they should be made of COTTON not nylon to allow the skin to breathe. Lying flat most of the day WILL PREVENT VARICOSITIES but will not be helpful for the client's overall health and function. Raising the legs and putting it against the wall will still create pressure in the legs.

9. In a 32 day menstrual cycle, ovulation usually occurs on the [2] A. 14th day after menstruation B. 18th day after menstruation C. 20th day after menstruation D. 24th day after menstruation * To get the day of ovulation, A diary is made for around 6 months to determine the number of days of menstrual cycle [ from onset of mens to the next onset of mens ] and the average is taken from that cycles. 14 days are subtracted from the total days of the menstrual cycle. This signifies the ovulation day. A couple would abstain having sex 5 days before and 5 days after the ovulation day. Therefore, a 32 day cycle minus 14 days equals 18, hence... ovulation occurs at the 18th day. 10. Placenta is the organ that provides exchange of nutrients and waste products between mother and fetus. This develops by [4] A. First month B. Third month C. Fifth month D. Seventh month * The placenta is formed at around 3 months. It is a latin word for PANCAKE because of it's appearance. It arises from the trophoblast from the chorionic villi and decidua basalis. It functions as the fetal lungs, kidney, GI tract and an endocrine organ. 11. In evaluating the weight gain of Aling Julia, you know the minimum weight gain during pregnancy is [3] A. 2 lbs/wk B. 5 lbs/wk C. 7 lbs/wk D. 10 lbs/wk * Weight gain should be 1 to 2 lbs per week during the 2nd and 3rd trimester and 3 to 5 lbs gain during the first trimester for a total of 25-35 lbs recommended weight gain during the gravida state. 12. The more accurate method of measuring fundal height is [2] A. Millimeter B. Centimeter C. Inches D. Fingerbreadths

* Fundal height is measured in cm not mm. centimeters is the more accurate method of measuring fundic height than inches or fingerbreadths. 13. To determine fetal position using Leopold's maneuvers, the first maneuver is to [1] A. Determine degree of cephalic flexion and engagement B. Determine part of fetus presenting into pelvis C. Locate the back,arms and legs D. Determine what part of fetus is in the fundus * Leopold's one determines what is it in the fundus. This determines whether the fetal head or breech is in the fundus. A head is round and hard. Breech is less well defined. 14. Aling julia has encouraged her husband to attend prenatal classes with her. During the prenatal class, the couple expressed fear of pain during labor and delivery. The use of touch and soothing voice often promotes comfort to the laboring patient. This physical intervention is effective because [2] A. Pain perception is interrupted B. Gate control fibers are open C. It distracts the client away from the pain D. Empathy is communicated by a caring person * Touch and soothing voice promotes pain distraction. Instead of thinking too much of the pain in labor, The mother is diverted away from the pain sensation by the use of touch and voice. Pain perception is not interrupted, pain is still present. When gate control fibers are open, Pain is felt according to the gate control theory of pain. Although empathy is communicated by the caring person, this is not the reason why touch and voice promotes comfort to a laboring patient. 15. Which of the following could be considered as a positive sign of pregnancy ? [1] A. Amenorrhea, nausea, vomiting B. Frequency of urination C. Braxton hicks contraction D. Fetal outline by sonography * Fetal outline by sonography or other imaging devices is considered a positive sign of pregnancy along with the presence of fetal heart rate and movement felt by a qualified examiner. All those signs with the discoverer's name on them [ chadwick, hegars, braxton hicks, goodells ] are considered probable and All the physiologic changes brought about by pregnancy like hyperpigmentation, fatgiue, uterine enlargement, nausea, vomiting, breast changes, frequent urination are considered presumptive. Sonographic evidence of the gestational sac is not POSITIVE sign but rather, PROBABLE.

SITUATION : [FFC] Maternal and child health is the program of the department of health created to lessen the death of infants and mother in the philippines. [2] 16. What is the goal of this program? A. Promote mother and infant health especially during the gravida stage B. Training of local hilots C. Direct supervision of midwives during home delivery D. Health teaching to mother regarding proper newborn care * The goal of the MCHN program of the DOH is the PROMOTION AND MAINTENANCE OF OPTIMUM HEALTH OF WOMAN AND THEIR NEWBORN. To achieve this goal, B,C and D are all carried out. Even without the knowledge of the MCHN goal you SHOULD answer this question correctly. Remember that GOALS are your plans or things you MUST ATTAIN while STRATEGIES are those that must be done [ ACTIONS ] to attain your goal. Looking at B,C and D they are all ACTIONS. Only A correctly followed the definition of a goal. 17. One philosophy of the maternal and child health nursing is [1] A. All pregnancy experiences are the same for all woman B. Culture and religious practices have little effect on pregnancy of a woman C. Pregnancy is a part of the life cycle but provides no meaning D. The father is as important as the mother * Knowing that not all individuals and pregnancy are the same for all women, you can safely eliminate letter A. Personal, culture and religious attitudes influence the meaning of pregnancy and that makes pregnancy unique for each individual. Culture and religious practice have a great impact on pregnancy, eliminate B. Pregnancy is meaningful to each individuals, not only the mother but also the father and the family and the father of the child is as important as the mother. MATERNAL AND CHILD HEALTH IS FAMILY CENTERED and thid will guide you in correctly answering D. 18. In maternal care, the PHN responsibility is [2] A. To secure all information that would be needing in birth certificate B. To protect the baby against tetanus neonatorum by immunizing the mother with DPT C. To reach all pregnant woman D. To assess nutritional status of existing children * The sole objective of the MCHN of the DOH is to REACH ALL PREGNANT WOMEN AND GIVE SUFFICIENT CARE TO ENSURE A HEALTHY PREGNANCY AND THE BIRTH OF A FULL TERM HEALTH BABY. As not to confuse this with the GOAL of the MCHN, The OBJECTIVE should answer the GOAL, they are different. GOAL : to promote and maintain optimum health for women and their newborn HOW? OBJECTIVE : By reaching all pregnant

women to give sufficient care ensuring healthy pregnancy and baby. 19. This is use when rendering prenatal care in the rural health unit. It serves as a guide in Identification of risk factors [1] A. Underfive clinic chart B. Home based mother's record C. Client list of mother under prenatal care D. Target list of woman under TT vaccination * The HBMR is used in rendring prenatal care as guide in identifying risk factors. It contains health promotion message and information on the danger signs of pregnancy. 20. The schedule of prenatal visit in the RHU unit is [4] A. Once from 1st up to 8th month, weekly on the 9th month B. Twice in 1st and second trimester, weekly on third trimester C. Once in each trimester, more frequent for those at risk D. Frequent as possible to determine the presence of FHT each week * Visit to the RHU should be ONCE each trimester and more frequent for those who are high risks. The visit to the BHS or health center should be ONCE for 1st to 6th months of pregnancy, TWICE for the 7th to 8th month and weekly during the 9th month. They are different and are not to be confused with. SITUATION : Knowledge of the menstrual cycle is important in maternal health nursing. The following questions pertains to the process of menstruation 21. Menarche occurs during the pubertal period, Which of the following occurs first in the development of female sex characteristics? [2] A. Menarche B. Accelerated Linear Growth C. Breast development D. Growth of pubic hair * Remember TAMO or THELARCHE, ADRENARCHE, MENARCHE and OVULATION. Telarche is the beginning of the breast development which is influenced by the increase in estrogen level during puberty. Adrenarche is the development of axillary and pubic hair due to androgen stimulation. Menarche is the onset of first menstruation that averagely occurs at around 12 to 13 years old. Ovulation then occurs last. However, prior to TAMO, Accelerated LINEAR GROWTH will occur first in GIRLS while WEIGHT INCREASE is the first one to occur in boys. 22. Which gland is responsible for initiating the menstrual cycle? [3]

A. Ovaries B. APG C. PPG D. Hypothalamus * Hypothalamus secretes many different hormones and one of them is the FSHRF or the FOLLICLE STIMULATING HORMONE RELEASING FACTOR. This will instruct the ANTERIOR PITUITARY GLAND to secrete FSH that will stimulate the ovary to release egg and initiate the menstrual cycle. The PPG or the posterior pituitary only secretes two hormones : OXYTOCIN and ADH. It plays an important factor in labor as well as in the pathophysiology of diabetes insipidus. 23. The hormone that stimulates the ovaries to produce estrogen is [1] A. GnRH B. LH C. LHRF D. FSH * FSH stimulates the ovaries to secrete estrogen. This hormone is a 3 substance compounds known as estrone [e1], estradiol [2] and estriol [3] responsible for the development of female secondary sex characteristics. It also stimulates the OOCYTES to mature. During pregnancy, Estrogen is secreted by the placenta that stimulates uterine growth to accomodate the fetus. 24. Which hormone stimulates oocyte maturation? [2] A. GnRH B. LH C. LHRF D. FSH * Refer to #23 25. When is the serum estrogen level highest in the menstrual cycle? [4] A. 3rd day B. 13th day C. 14th day D. End of menstrual cycle * There are only 3 days to remember in terms of hormonal heights during pregnancy. 3,13 and 14. During the 3rd day, Serum estrogen is the lowest. During the 13th day, Serum estrogen is at it's peak while progestrone is at it's lowest and this signifies that a mature oocyte is ready for release. At 14th day, Progesterone will surge and this is the reason why

there is a sudden increase of temperature during the ovulation day and sudden drop during the previous day. This will not stimulate the release of the mature egg or what we call, OVULATION.

26. To correctly determine the day of ovulation, the nurse must [2] A. Deduct 14 days at the mid of the cycle B. Subtract two weeks at cycle's end C. Add 7 days from mid of the cycle D. Add 14 days from the end of the cycle * Refer to # 9 27. The serum progesterone is lowest during what day of the menstrual cycle? [4] A. 3rd day B. 13th day C. 14th day D. End of menstrual cycle * At 3rd day, The serum estrogen is at it's lowest. At the 13th day, serum estrogen is at it's peak while progesterone is at it's lowest. At the 13th day of the cycle, An available matured ovum is ready for fertilization and implantation. The slight sharp drop of temperature occurs during this time due to the very low progestrone level. The next day, 14th day, The serum progestrone sharply rises and this causes the release of the matured ovum. Temperature also rises at this point because of the sudden increase in the progestrone level. 28. How much blood is loss on the average during menstrual period? [4] A. Half cup B. 4 tablespoon C. 3 ounces D. 1/3 cup * The average blood loss during pregnancy is 60 cc. A, half cup is equivalent to 120 cc. C, is equivalent to 90 cc while D, is equivalent to 80 cc. 1 tablespoon is equal to 15 ml. 4 tablespoon is exactly 60 cc. 29. Menstruation occurs because of which following mechanism? [2] A. Increase level of estrogen and progesterone level B. Degeneration of the corpus luteum C. Increase vascularity of the endothelium D. Surge of hormone progesterone

* Degeneration of the corpus luteum is the cause of menstruation. Menstruation occurs because of the decrease of both estrogen and progestrone. This is caused by the regression of the corpus luteum inside the ovary 8 to 10 days in absence of fertilization after an ovum was released. With the absence of progestrone, the endometrium degenerates and therefore, vascularity will decrease at approximately 25th day of the cycle which causes the external manifestation of menstruation. 30. If the menstrual cycle of a woman is 35 day cycle, she will approximately [2] A. Ovulate on the 21st day with fertile days beginning on the 16th day to the 26th day of her cycle B. Ovulate on the 21st day with fertile days beginning on the 16th day to the 21th day of her cycle C. Ovulate on the 22st day with fertile days beginning on the 16th day to the 26th day of her cycle D. Ovulate on the 22st day with fertile days beginning on the 14th day to the 30th day of her cycle * Formula for getting the fertile days and ovulation day is : Number of days of cycle MINUS 14 [ Ovulation day ] Minus 5 Plus 5 [ Possible fertile days ]. Since the client has a 35 day cycle, we subtract 14 days to get the ovulation day which is 21. Minus 5 days is equal to [21 - 5 = 16 ] 16 , Plus 5 days [ 21 + 5 = 26 ] is equal to 26. Therefore, Client is fertile during the 16th to the 26th day of her cycle. This is the same principle and formula used in the calendar / rhythm method. SITUATION : Wide knowledge about different diagnostic tests during pregnancy is an essential arsenal for a nurse to be successful. 31. The Biparietal diameter of a fetus is considered matured if it is atleast [4] A. 9.8 cm B. 8.5 cm C. 7.5 cm D. 6 cm * BPD is considered matured at 8.5 cm and at term when it reaches 9.6 cm. 32. Quickening is experienced first by multigravida clients. At what week of gestation do they start to experience quickening? [4] A. 16th B. 20th C. 24th D. 28th

* Multigravid clients experience quickening at around 16 weeks or 4 months. Primigravid clients experience this 1 month later, at the 5th month or 20th week. 33. Before the start of a non stress test, The FHR is 120 BPM. The mother ate the snack and the practitioner noticed an increase from 120 BPM to 135 BPM for 15 seconds. How would you read the result? [3] A. Abnormal B. Non reactive C. Reactive D. Inconclusive, needs repeat * Normal non stress test result is REACTIVE. Non stress test is a diagnostic procedure in which the FHR is compared with the child's movement. A normal result is an increase of 15 BPM sustained for 15 seconds at every fetal movement. The mother is told to eat a light snack during the procedure while the examiner carefully monitors the FHR. The mother will tell the examiner that she felt a movement as soon as she feels it while the examiner take note of the time and the FHR of the fetus. 34. When should the nurse expect to hear the FHR using a fetoscope? [4] A. 2nd week B. 8th week C. 2nd month D. 4th month * The FHR is heard at about 4 months using a fetoscope. Remember the word FeFOUR to relate fetoscope to four. 35. When should the nurse expect to hear FHR using doppler Ultrasound? [4] A. 8th week B. 8th month C. 2nd week D. 4th month * The FHR is heard as early as 8th week [ some books, 12 to 14 weeks ] using doppler ultrasound. Remember the word DOPPLE RATE, [ DOPPLER 8 ] to relate dopple ultrasound to the number 8. 36. The mother asks, What does it means if her maternal serum alpha feto protein is 35 ng/ml? The nurse should answer [4] A. It is normal B. It is not normal C. 35 ng/ml indicates chromosomal abberation

D. 35 ng/ml indicates neural tube defect * The normal maternal alpha feto protein is 38-45 ng/ml. Less 38 than this indicates CHROMOSOMAL ABBERATION [Down,Klinefelters] and more than 45 means NEURAL TUBE DEFECTS [Spina Bifida]. Remember the word CLINICAL NURSE. C for chromosomal abberation for <38>N for neural tube defect for >45. C<38>45 Clinic Nurse. CLINIC NURSE is also an important mnemonics to differentiate COUNTER TRANSFERENCE from TRANSFERENCE. Counter transference is the special feeling of the CLINIC NURSE or CLINICIAN to the patient while transference is the development of personal feelings of the patient to the nurse. 37. Which of the following mothers needs RHOGAM? [1] A. RH + mother who delivered an RH - fetus B. RH - mother who delivered an RH + fetus C. RH + mother who delivered an RH + fetus D. RH - mother who delivered an RH - fetus * Rhogam is given to RH - Mothers That delivers an RH + Fetus. Rhogam prevents ISOIMMUNIZATION or the development of maternal antibodies against the fetal blood due to RH incompatibility. Once the mother already develops an antibody against the fetus, Rhogam will not anymore be benificial and the mother is advised no to have anymore pregnancies. Rhogam is given within 72 hours after delivery. 38. Which family planning method is recommended by the department of health more than any other means of contraception? [4] A. Fertility Awareness Method B. Condom C. Tubal Ligation D. Abstinence * Abstinence is never advocated as a family planning method. Though, It is probably the BEST METHOD to prevent STD and pregnancy, it is inhumane and supresses the reproductive rights of the people. It is also unrealistic. FAM is advocated by the DOH more than any other kind of contraception. It is a combination of symptothermal and billings method. CALENDAR method is the only method advocated by the catholic church. 39. How much booster dose does tetanus toxoid vaccination for pregnant women has? [4] A. 2 B. 5 C. 3 D. 4

* TT1 and TT2 are both primary dosages. While TT3 up to TT5 represents the booster dosages. 40. Baranggay pinoybsn.tk has 70,000 population. How much nurse is needed to service this population? [4] A. 5 B. 7 C. 50 D. 70 * For every 10,000 population , 1 nurse is needed. therefore, a population of 70,000 people needs a service of 7 nurses. SITUATION : [ND2I246] Reproductive health is the exercise of reproductive right with responsibility. A married couple has the responsibility to reproduce and procreate. 41. Which of the following is ONE of the goals of the reproductive health concept? [3] A. To achieve healthy sexual development and maturation B. To prevent specific RH problem through counseling C. Provide care, treatment and rehabilitation D. To practice RH as a way of life of every man and woman * EVERY ACHIEVER AVOIDS RECEIVER : Remember this mnemonics and it will guide you in differentiating which is which from the goals, visions and strategies. If a sentence begins with these words, it is automatically a GOAL. Usually, The trend in the board is that they will mix up the vision, strategies and goals to confuse you. D is the only vision of the RH program. Anything else aside from the vision and goals are more likely strategies. [ B and C ] Strategies, even without knowing them or memorizing them can easily be seperated as they convey ACTIONS and ACTUAL INTERVENTIONS. This is universal and also applies to other DOH programs. Notice that B and C convey actions and interventions. 42. Which of the following is NOT an element of the reproductive health? [4] A. Maternal and child health and nutrition B. Family planning C. Prevention and management of abortion complication D. Healthy sexual development and nutrition * Achieving healthy sexual development and nutrition is a GOAL of the RH. Knowledge of the elements, goals, strategies and vision of RH are important in answering this question. I removed the word ACHIEVE to let you know that it is possible for the board of nursing not to include those keywords [ although it never happened as of yet ].

43. In the international framework of RH, which one of the following is the ultimate goal? [3] A. Women's health in reproduction B. Attainment of optimum health C. Achievement of women's status D. Quality of life * Quality of life is the ultimate goal of the RH in the international framework. Way of life is the ultimate goal of RH in the local framework. 44. Which one of the following is a determinant of RH affecting woman's ability to participate in social affairs? [3] A. Gender issues B. Socio-Economic condition C. Cultural and psychosocial factors D. Status of women * This is an actual board question, Gender issues affects the women participation in the social affairs. Socio economic condition is the determinant for education, employment, poverty, nutrition, living condition and family environment. Status of women evolves in women's rights. Cultural and psychosocial factors refers to the norms, behaviors, orientation, values and culture. Refer to your DOH manual to read more about this. 45. In the philippine RH Framework. which major factor affects RH status? [4] A. Women's lower level of literacy B. Health service delivery mechanism C. Poor living conditions lead to illness D. Commercial sex workers are exposed to AIDS/STD. * Health services delivery mechanism is the major factor that affect RH status. Other factors are women's behavior, Sanitation and water supply, Employment and working conditions etc. 46. Which determinant of reproductive health advocates nutrition for better health promotion and maintain a healthful life? [4] A. Socio-Economic conditions B. Status of women C. Social and gender issues D. Biological, Cultural and Psychosocial factors * Refer to # 44

47. Which of the following is NOT a strategy of RH? [3] A. Increase and improve contraceptive methods B. Achieve reproductive intentions C. Care provision focused on people with RH problems D. Prevent specific RH problem through information dessemination * Refer to #41 48. Which of the following is NOT a goal of RH? [3] A. Achieve healthy sexual development and maturation B. Avoid illness/diseases, injuries, disabilities related to sexuality and reproduction C. Receive appropriate counseling and care of RH problems D. Strengthen outreach activities and the referral system * Refer to #41 49. What is the VISION of the RH? [2] A. Attain QUALITY OF LIFE B. Practice RH as a WAY OF LIFE C. Prevent specific RH problem D. Health in the hands of the filipino * Refer to #43 SITUATION : [SORANGE19] Baby G, a 6 hours old newborn is admitted to the NICU because of low APGAR Score. His mother had a prolonged second stage of labor 50. Which of the following is the most important concept associated with all high risk newborn? [1] A. Support the high-risk newborn's cardiopulmonary adaptation by maintaining adequate airway B. Identify complications with early intervention in the high risk newborn to reduce morbidity and mortality C. Assess the high risk newborn for any physical complications that will assist the parent with bonding D. Support mother and significant others in their request toward adaptation to the high risk newborn * The 3 major and initial and immediate needs of newborns both normal and high risks are AIR/BREATHING, CIRCULATION and TEMPERATURE. C and D are both eliminated because they do not address the immediate newborn needs. Identifying complication with

early intervention is important, however, this does not address the IMMEDIATE and MOST IMPORTANT newborn needs. 51. Which of the following would the nurse expect to find in a newborn with birth asphyxia? [1] A. Hyperoxemia B. Acidosis C. Hypocapnia D. Ketosis * Birth Asphyxia is a term used to describe the inability of an infant to maintain an adequate respiration within 1 minute after birth that leads so acidosis, hypoxia, hypoxemia and tissue anoxia. This results to Hypercapnia not Hypocapnia due to the increase in carbonic acid concentration in the fetal circulation because the carbon dioxide fails to get eliminated from the infant's lungs because of inadequate respiration. Ketosis is the presence of ketones in the body because of excessive fat metabolism. This is seen in diabetic ketoacidosis. 52. When planning and implementing care for the newborn that has been successfully resuscitated, which of the following would be important to assess? [1] A. Muscle flaccidity B. Hypoglycemia C. Decreased intracranial pressure D. Spontaneous respiration * There is no need to assess for spontaneous respiration because OF the word SUCCESSFULLY RESUSCITATED. What is it to assess is the quality and quantity of respiration. Infants who undergone tremendous physical challenges during birth like asphyxia, prolonged labor, RDS are all high risk for developing hypoglycemia because of the severe depletion of glucose stores to sustain the demands of the body during those demanding times. SITUATION : [P-I/46] Nurses should be aware of the different reproductive problems. 53. When is the best time to achieve pregnancy? [2] A. Midway between periods B. Immediately after menses end C. 14 days before the next period is expected D. 14 days after the beginning of the next period * The best time to achieve pregnancy is during the ovulation period which is about 14 days before the next period is expected. A Menstrual cycle is defined as the number of days from the start of the menstruation period, up to the start of another menstrual period. To obtain

the ovulation day, Subtract 14 days from the end of each cycle. Example, The start of the menstrual flow was July 12, 2006. The next flow was experienced August 11, 2006. The length of the menstrual cycle is then 30 days [ August 11 minus July 12 ]. We then subtract 14 days from that total length of the cycle and that will give us 16 days [ 30 minus 14 ] Count 16 days from July 12, 2006 and that will give us July 28, 2006 as the day of ovulation. [ July 12 + 16 days ] This is the best time for coitus if the intention is getting pregnant, worst time if not. 54. A factor in infertility maybe related to the PH of the vaginal canal. A medication that is ordered to alter the vaginal PH is: [2] A. Estrogen therapy B. Sulfur insufflations C. Lactic acid douches D. Na HCO3 Douches * Sperm is innately ALKALINE. Too much acidity is the only PH alteration in the vagina that can kill sperm cells. Knowing this will direct you to answering letter D. Sodium Bicarbonate douches will make the vagina less acidic because of it's alkaline property, making the vagina's environment more conducive and tolerating to the sperm cells. Estrogen therapy will not alter the PH of the vaginal canal. HRT [ Hormone replacement therapy ] is now feared by many women because of the high risk in acquiring breast, uterine and cervical cancer. Research on this was even halted because of the significant risk on the sample population. Lactic acid douches will make the vagina more acidic, further making the environment hostile to the alkaline sperm. Sulfur insufflation is a procedure used to treat vaginal infections. A tube is inserted in the vagina and sulfur is introduced to the body. The yeasts, fungi and other microorganisms that are sensitive to sulfur are all immediately killed by it on contact. 55. A diagnostic test used to evaluate fertility is the postcoital test. It is best timed [2] A. 1 week after ovulation B. Immediately after menses C. Just before the next menstrual period D. Within 1 to 2 days of presumed ovulation * A poscoital test evaluates both ovulation detection and sperm analysis. When the woman ovulates [ by using the FAM method or commercial ovulation detection kits, woman should know she ovulates ] The couple should have coitus and then, the woman will go to the clinic within 2 to 8 hours after coitus. The woman is put on a lithotomy position. A specimen for cervical mucus is taken and examined for spinnbarkeit [ ability to stretch 15 cm before breaking ] and sperm count. Postcoital test is now considered obsolete because a single sperm and cervical mucus analysis provides more accurate data. 56. A tubal insufflation test is done to determine whether there is a tubal obstruction.

Infertility caused by a defect in the tube is most often related to a [3] A. Past infection B. Fibroid Tumor C. Congenital Anomaly D. Previous injury to a tube * PID [ Most common cause of tubal obstruction ] due to untreated gonorrhea, chlamydia or other infections that leads to chronic salphingitis often leads to scarring of the fallopian tube thereby causing tubal obstuction. This one of the common cause of infertility, the most common is Anovulation in female and low sperm count in males. A ruptured appendix, peritonitis and abdominal surgery that leads to infection and adhesion of the fallopian tube can also lead to tubal obstruction. 57. Which test is commonly used to determine the number, motility and activity of sperm is the [2] A. Rubin test B. Huhner test C. Friedman test D. Papanicolau test * Huhner test is synonymous to postcoital test. This test evaluates the number, motility and status of the sperm cells in the cervical mucus. refer to # 55 for more information. Rubin test is a test to determine the tubal patency by introducing carbon dioxide gas via a cannula to the client's cervix. The sound is then auscultated in the client's abdomen at the point where the outer end of the fallopian tube is located, near the fimbriae. Absent of sound means that the tube is not patent. Friedman test involves a FROG to determine pregnancy that is why it is also called as FROG TEST. Papanicolaou test [Correct spelling], discovered by Dr. George Papanicolaou during the 1930's is a cytolgic examination of the epithelial lining of the cervix. It is important in diagnosis cervical cancer. 58. In the female, Evaluation of the pelvic organs of reproduction is accomplished by [2] A. Biopsy B. Cystoscopy C. Culdoscopy D. Hysterosalpingogram * Biopsy is acquiring a sample tissue for cytological examination. Usually done in cancer grading or detecting atypical, abnormal and neoplastic cells. Cystoscopy is the visualization of the bladder using a cystoscope. This is inserted via the urethra. TURP or the transurethral resection of the prostate is frequently done via cystoscopy to remove the need for incision in resecting the enlarged prostate in BPH. Culdoscopy is the insertion of the culdoscope through the posterior vaginal wall between the rectum and uterus to visualize the douglas cul de sac. This is an important landmark because this is the lowest point in the pelvis, fluid

or blood tends to collect in this place. Hysterosalpingogram is the injection of a blue dye, or any radio opaque material through the cervix under pressure. X ray is then taken to visualize the pelvic organs. This is done only after menstruation to prevent reflux of the menstrual discharge up into the fallopian tube and to prevent an accidental irradiation of the zygote. As usual, as with all other procedures that ends in GRAM, assess for iodine allergy. 59. When is the fetal weight gain greatest? [3] A. 1st trimester B. 2nd trimester C. 3rd trimester D. from 4th week up to 16th week of pregnancy * Vital organs are formed during the first trimester, The greatest LENGTH gain occurs during the second trimester while the greatest weight gain occurs during the last trimester. This is the time when brown fats starts to be deposited in preparation for the upcoming delivery. 60. In fetal blood vessel, where is the oxygen content highest? [3] A. Umbilical artery B. Ductus Venosus C. Ductus areteriosus D. Pulmonary artery * Ductus venosus is directly connected to the umbilical vein, Which is directly connected to the highly oxygenated placenta. This vessel supplies blood to the fetal liver. Umbilical arteries carries UNOXYGENATED BLOOD, they carry the blood away from the fetal body. Ductus arteriosus shunts the blood away from the fetal lungs, this carries an oxygenated blood but not as concentrated as the blood in the ductus venosus who have not yet service any of the fetal organ for oxygen except the liver. Knowing that the fetal lungs is not yet functional and expanded will guide you to automatically eliminate the pulmonary artery which is responsible for carrying UNOXYGENATED BLOOD away from the lungs. 61. The nurse is caring for a woman in labor. The woman is irritable, complains of nausea and vomits and has heavier show. The membranes rupture. The nurse understands that this indicates [1] A. The woman is in transition stage of labor B. The woman is having a complication and the doctor should be notified C. Labor is slowing down and the woman may need oxytocin D. The woman is emotionally distraught and needs assistance in dealing with labor * The clue to the answer is MEMBRANES RUPTURE. Membranes, as a rule, rupture at full dilation [ 10 cm ] unless ruptured by amniotomy or ruptured at an earlier time. The last of the mucus plug from the cervix is also released during the transition phase of labor. We call

that the OPERCULUM as signaled by a HEAVIER SHOW. During the transition phase, Cervix is dilated at around 8 to 10 cm and contractions reaches their peak of intensity occuring every 2 to 3 minutes with a 60 to 90 second duration. At the transition phase, woman also experiences nausea and vomiting with intense pain. This question is LIFTED from the previous board and the question was patterned EXACTLY WORD PER WORD from pillitteri. SITUATION : [J2I246] Katherine, a 32 year old primigravida at 39-40 weeks AOG was admitted to the labor room due to hypogastric and lumbo-sacral pains. IE revealed a fully dilated, fully effaced cervix. Station 0. 62. She is immediately transferred to the DR table. Which of the following conditions signify that delivery is near? [2] I - A desire to defecate II - Begins to bear down with uterine contraction III - Perineum bulges IV - Uterine contraction occur 2-3 minutes intervals at 50 seconds duration A. I,II,III B. I,II,III,IV C. I,III,IV D. II,III,IV * Again, lifted word per word from Pillitteri and this is from the NLE. A is the right answer. A woman near labor experiences desire to defecate because of the pressure of the fetal head that forces the stool out from the anus. She cannot help but bear down with each of the contractions and as crowning occurs, The perineum bulges. A woman with a 50 second contraction is still at the ACTIVE PHASE labor [ 40 to 60 seconds duration, 3 to 5 minutes interval ] Women who are about to give birth experience 60-90 seconds contraction occuring at 2-3 minutes interval. 63. Artificial rupture of the membrane is done. Which of the following nursing diagnoses is the priority? [2] A. High risk for infection related to membrane rupture B. Potential for injury related to prolapse cord C. Alteration in comfort related to increasing strength of uterine contraction D. Anxiety related to unfamiliar procedure * Nursing diagnosis is frequently ask. In any case that INFECTION was one of the choices, remove it as soon as you see it in ALL CASES during the intra and pre operative nursing care. Infection will only occur after 48 hours of operation or event. B is much more immediate and more likely to occur than A, and is much more FATAL. Prioritization and Appropriateness is the key in correctly answering this question. High risk for infection is an

appropriate nursing diagnosis, but as I said, Infection will occur in much later time and not as immediate as B. Readily remove D and C because physiologic needs of the mother and fetus take precedence over comfort measures and psychosocial needs. 64. Katherine complains of severe abdominal pain and back pain during contraction. Which two of the following measures will be MOST effective in reducing pain? [4] I - Rubbing the back with a tennis ball II- Effleurage III-Imagery IV-Breathing techniques A. II,IV B. II,III C. I,IV D. I,II * Remove B. Imagery is not used in severe pain. This is a labor pain and the mother will never try to imagine a nice and beautiful scenery with you at this point because the pain is all encompassing and severe during the transition phase of labor. Remove A and C Because breathing techniques is not a method to ELIMINATE PAIN but a method to reduce anxiety, improve pushing and prevent rapid expulsion of the fetus during crowning [ By PANTING ] Back pain is so severe during labor in cases of Posterior presentations [ ROP,LOP,RMP,LSaP, etc... ] Mother is asked to pull her knees towards her chest and rock her back. [ As in a rocking chair ] A Tennis ball rubbed at the client's back can relieve the pain due to the pressure of the presenting part on the posterior part of the birth canal. Also, rubbing a tennis ball to the client's back OPENS THE LARGE FIBER NERVE GATE. Effleurage or a simple rotational massage on the abdomen simply relieves the client's pain by opening the large fiber nerve gate and closing the the small fiber nerve gate. [ Please read about Gate control theory by Mezack and Wall ]. 65. Lumbar epidural anesthesia is administered. Which of the following nursing responsibilities should be done immediately following procedure? [1] A. Reposition from side to side B. Administer oxygen C. Increase IV fluid as indicated D. Assess for maternal hypotension * Hypotension is one of the side effects of an epidural anesthesia. An epidural anesthesia is injected on the L3 - L4 or L4 - L5 area. The injection lies just above the dura and must not cross the dura [ spinal anesthesia crosses the dura ]. Nursing intervention revolves in assesing RR, BP and other vital signs for possible complication and side effects. There is no need to position the client from side to side, The preferred position during the transition phase of labor is LITHOTOMY. Oxygen is not specific after administration of an epidural

anesthesia. IV fluid is not increased without doctor's order. AS INDICATED is different from AS ORDERED. 66. Which is NOT the drug of choice for epidural anesthesia? [4] A. Sensorcaine B. Xylocaine C. Ephedrine D. Marcaine * A,B and D are all drugs of choice for epidural anesthesia. Ephedrine is the drug use to reverse the symptom of hypotension caused by epidural anesthesia. It is a sympathomimetic agent that causes vasoconstriction, bronchodilation [ in asthma ] and can increase the amount of energy and alertness. Ephedrine is somewhat similar to epinephrine in terms of action as well as it's adverse effects of urinary retention, tremor, hypersalivation, dyspnea, tachycardia, hypertension. SITUATION : [SORANGE217] Alpha, a 24 year old G4P3 at full term gestation is brought to the ER after a gush of fluid passes through here vagina while doing her holiday shopping. 67. She is brought to the triage unit. The FHT is noted to be 114 bpm. Which of the following actions should the nurse do first? [2] A. Monitor FHT ever 15 minutes B. Administer oxygen inhalation C. Ask the charge nurse to notify the Obstetrician D. Place her on the left lateral position * Remove A. A FHR of 114 bpm is 6 beats below normal. Though monitoring is continuous and appropriate, This is not your immediate action. B, Oxygen inhalation needs doctor's order and therefore, is a DEPENDENT nursing action and won't be your first option. Although administration of oxygen by the nurse is allowed when given at the lowest setting during emergency situation. C is appropriate, but should not be your IMMEDIATE action. The best action is to place the client on the LEFT LATERAL POSITION to decrease the pressure in the inferior vena cava [ by the gravid uterus ] thereby increasing venus return and giving an adequate perfusion to the fetus. Your next action is to call and notify the obstetrician. Remember to look for an independent nursing action first before trying to call the physician. 68. The nurse checks the perineum of alpha. Which of the following characteristic of the amniotic fluid would cause an alarm to the nurse? [1] A. Greenish B. Scantly C. Colorless D. Blood tinged

* A greenish amniotic fluid heralds fetal distress not unless the fetus is in breech presentation and pressure is present on the bowel. Other color that a nurse should thoroughly evaluate are : Tea colored or strong yellow color that indicates hemolytic anemia , as in RH incompatibility. 69. Alpha asks the nurse. "Why do I have to be on complete bed rest? I am not comfortable in this position." Which of the following response of the nurse is most appropriate? [3] A. Keeping you on bed rest will prevent possible cord prolapse B. Completed bed rest will prevent more amniotic fluid to escape C. You need to save your energy so you will be strong enough to push later D. Let us ask your obstetrician when she returns to check on you * Once the membrane ruptures, as in the situation of alpha, The immediate and most appropriate nursing diagnosis is risk for injury related to cord prolapse. Keeping the client on bed rest is one of the best intervention in preventing cord prolapse. Other interventions are putting the client in a modified T position or Kneed chest position. Once the amniotic fluid escapes, It is allowed to escape. Although bed rest does saves energy, It is not the most appropriate response why bed rest is prescribed after membranes have ruptured. Not answering the client's question now will promote distrust and increase client's anxiety. It will also make the client think that the nurse is incompetent for not knowing the answer. 70. Alpha wants to know how many fetal movements per hour is normal, the correct response is [4] A. Twice B. Thrice C. Four times D. 10-12 times * According to Sandovsky, To count for the fetal movement, Mother is put on her LEFT SIDE to decrease placental insufficiency. This is usually done after meals. The mother is asked to record the number of fetal movements per hour. A fetus moves Twice every 10 minutes and 10 to 12 times times an hour. In SIA'S Book, She answered this question with letter B. But according to Pillitteri, A movement fewer than 5 in an hour is to be reported to the health care provider. The Board examiners uses Pillitteri as their reference and WORD PER WORD, Their question are answered directly from the Pillitteri book. 10-12 times according to Pillitteri, is the normal fetal movement per minute.

71. Upon examination by the obstetrician, he charted that Alpha is in the early stage of labor. Which of the following is true in this state? [1] A. Self-focused

B. Effacement is 100% C. Last for 2 hours D. Cervical dilation 1-3 cm * The earliest phase of labor is the first stage of labor : latent phase characterized by a cervical dilation of 0-3 cm, Mild contraction lasting for 20 to 40 seconds. This lasts approximately 6 hours in primis and 4.5 hours in multis. C is the characteristic of ACTIVE PHASE of labor, Characterized by a cervical dilation of 4-7 cm and contractions of 40 to 60 seconds. This phase lasts at around 3 hours in primis and 2 hours in multis. Effacement of 100% is a characteristic of the TRANSITION PHASE as well as being self focused.

SITUATION : Maternal and child health nursing a core concept of providing health in the community. Mastery of MCH Nursing is a quality all nurse should possess. 72. When should be the 2nd visit of a pregnant mother to the RHU? [2] A. Before getting pregnant B. As early in pregnancy C. Second trimester D. Third trimester * Visit to the RHU are once every trimester and more frequent for those women at risk. Visit to the health center is once during the 0-6th month of pregnancy, twice during the 7th-8th month and weekly at the last trimester.

73. Which of the following is NOT a standard prenatal physical examination? [1] A. Neck examination for goiter B. Examination of the palms of the hands for pallor C. Edema examination of the face hands, and lower extremeties D. Examination of the legs for varicosities 74. Which of the following is NOT a basic prenatal service delivery done in the BHS? [2] A. Oral / Dental check up B. Laboratory examination C. Treatment of diseases D. Iron supplementation * A is done at the RHU not in BHS. 75. How many days and how much dosage will the IRON supplementation be taken? [4] A. 365 days / 300 mg

B. 210 days / 200 mg C. 100 days/ 100mg D. 50 days / 50 mg * Iron supplementation is taken for 210 days starting at the 5th month of pregnancy up to 2nd month post partum. Dosage can range from 100 to 200 mg. 76. When should the iron supplementation starts and when should it ends? [4] A. 5th month of pregnancy to 2nd month post partum B. 1st month of pregnancy to 5th month post partum C. As early in pregnancy up to 9th month of pregnancy D. From 1st trimester up to 6 weeks post partum * Refer to #75 77. In malaria infested area, how is chloroquine given to pregnant women? [4] A. 300 mg / twice a month for 9 months B. 200 mg / once a week for 5 months C. 150 mg / twice a week for the duration of pregnancy D. 100 mg / twice a week for the last trimester of pregnancy * Always remember that chloroquine is given twice a week for the whole duration of pregnancy. This knowledge alone will lead you to correctly identifying letter C. 78. Which of the following mothers are qualified for home delivery? [2] A. Pre term B. 6th pregnancy C. Has a history of hemorrhage last pregnancy D. 2nd pregnancy, Has a history of 20 hours of labor last pregnancy. * Knowing that a preterm mother is not qualified for home delivery will help you eliminate A. History of complications like bleeding, CPD, Eclampsia and diseases like TB, CVD, Anemia also nulls this qualification. A qualified woman for home delivery should only had less than 5 pregnancies. More than 5 disqualifies her from home delivery. High risk length of labor for primigravidas ls more than 24 hours and for multigravidas, it is more than 12 hours. Knowing this will allow you to choose D. 79. Which of the following is not included on the 3 Cs of delivery? [2] A. Clean Surface B. Clean Hands C. Clean Equipments D. Clean Cord

* 3 Cs of delivery are CLEAN SURFACE,HANDS AND CORD. " Kinamay ni Cordapya ang labada gamit ang Surf - Budek " 80. Which of the following is unnecessary equipment to be included in the home delivery kit? [4] A. Boiled razor blade B. 70% Isopropyl Alcohol C. Flashlight D. Rectal and oral thermometer * Home delivery kit should contain the following : Clamps, Scissors, Blade, Antiseptic, Soap and hand brush, Bp app, Clean towel or cloth and Flashlight. Optional equipments include : Plastic sheet, Suction bulb, Weighing scale, Ophthalmic ointment, Nail cutter, Sterile gloves, Rectal and oral thermometers.

SITUATION : [NBLUE166] Pillar is admitted to the hospital with the following signs : Contractions coming every 10 minutes, lasting 30 seconds and causing little discomfort. Intact membranes without any bloody shows. Stable vital signs. FHR = 130bpm. Examination reveals cervix is 3 cm dilated with vertex presenting at minus 1 station. 81. On the basis of the data provided above, You can conclude the pillar is in the [1] A. In false labor B. In the active phase of labor C. In the latent phase of labor D. In the transitional phase of labor * Refer to #71 82. Pitocin drip is started on Pilar. Possible side effects of pitocin administration include all of the following except [3] A. Diuresis B. Hypertension C. Water intoxication D. Cerebral hemorrhage * Oxytocin [ Pitocin ] is a synthetic form of hormone naturally released by the PPG. It is used to augment labor and delivery. Dosage is about 1 to 2 milli units per minute and this can be doubled until the desired contraction is met. Side effects are Water intoxication, Diuresis, Hypertonicity of the uterus, Uterine rupture, Precipitated labor, Walang kamatayang Nausea and Vomiting and Fetal bradycardia. Diuresis occurs because of

water intoxication, The kidney will try to compensate to balance the fluid in the body. NEVER give pitocin when FHR is below 120. Even without knowing anything about Pitocin, A cerebral hemorrhage is LETHAL and DAMAGE IS IRREVERSIBLE and if this is a side effect of a drug, I do not think that FDA or BFAD will approve it. 83. The normal range of FHR is approximately [3] A. 90 to 140 bpm B. 120 to 160 bpm C. 100 to 140 bpm D. 140 to 180 bpm * A normal fetal heart rate is 120-160 bpm. 84. A negative 1 [-1] station means that [1] A. Fetus is crowning B. Fetus is floating C. Fetus is engaged D. Fetus is at the ischial spine * At the negative station, The fetus is not yet engaged and floating. At 0 station, it means that the fetus is engaged to the ischial spine. Crowning occurs when the fetus is at the +3,+4 Station. Stations signifies distance of the presensting part below or above ischial spine. + denoted below while - denotes above. The number after the sign denotes length in cm. +1 station therefore means that the presenting part is 1 cm below the ischial spine. 85. Which of the following is characteristics of false labor [1] A. Bloody show B. Contraction that are regular and increase in frequency and duration C. Contraction are felt in the back and radiates towards the abdomen D. None of the above * A,B and C are all charactertistics of a true labor. True labor is heralded by LIGHTENING. This makes the uterus lower and more anterior. This occrs 2 weeks prior to labor. At the morning of labor, women experiences BURST OF ENERGY because of adrenaline rush induced by the decrease progestrone secretion of the deteriorating placenta. The pain in labor is felt at the back and radiates towards the abdomen and becomes regular, increasing frequency and duration. As the cervix softens and dilates, The OPERCULUM or the mucus plug is expelled. False labor is characterized by Irregular uterine contraction that is relieved by walking, Pain felt at the abdomen and confined there and in the groin, The cervix do not achieve dilation and Pain that is relieved by sleep and do not increase in intensity and duration.

86. Who's Theory of labor pain that states that PAIN in labor is cause by FEAR [4] A. Bradley B. Simpson C. Lamaze D. Dick-Read * Believe it or not, this is an actual board question. Grantley Dick-Read is just one person. Usually a two name theory means two theorist. He published a book in 1933 "CHILDBIRTH WITHOUT FEAR". He believes that PAIN in labor is caused by FEAR that causes muscle tension, thereby halting the blood towards the uterus and causing decreased oxygenation which causes the PAIN. 1950s French obstetrician, Dr. Ferdinand Lamaze perhaps is the most popular theorist when it comes to labor. The theory behind Lamaze is that birth is a normal, natural and healthy event that should occur without unnecessary medical intervention. Rather than resorting to pain medication, different breathing techniques are used for each stage of labor to control pain. Fathers are assigned the role of labor coach, and are responsible for monitoring and adjusting their partner's breathing pattern throughout childbirth. In 1965, obstetrician Robert A. Bradley, MD wrote "Husband Coached Childbirth." The Bradley method perhaps is the easiest to remember, BRAD ley necessitates the presence of the FATHER during labor. Bradley Method views birth as a natural process. This method also emphasizes the importance of actively involving fathers in the labor process. Fathers are taught ways to help ease their partner's pain during childbirth through guided relaxation and slow abdominal breathing. James Young Simpson is an english doctor and the first to apply anesthesia during labor and child birth. He uses ETHER to alleviate labor pain. He then discovered the effects of chloroform as an anesthetic agent. Because of his works, He was recognized by Queen Victoria because the queen herself uses Simpson's chloroform in alleviating labor pain when she gave birth to prince leopold. 87. Which sign would alert the nurse that Pillar is entering the second stage of labor? [1] A. Increase frequency and intensity of contraction B. Perineum bulges and anal orifice dilates C. Effacement of internal OS is 100% D. Vulva encircles the largest diameter of presenting part * The second stage of labor begins as the cervical internal os is 100% effaced and fully dilated. It ends after the fetus has been delivered. Crowning, as in letter B and D is too late of a sign to alert the nurse that Pillar is entering the second stage of labor. A occurs during the first stage of labor.

88. Nursing care during the second stage of labor should include [1] A. Careful evaluation of prenatal history B. Coach breathing, Bear down with each contraction and encourage patient. C. Shave the perineum D. Administer enema to the patient * The second stage of labor begins with a full cervical dilation and effacement and finishes when the baby is fully delivered. Careful evaluation of prenatal history is done on admission and check ups and is never done in the second stage of labor. Shaving the perineum and enema are done during the first stage of labor in preparation for delivery or before labor begins when client is admitted. Enema is not a routine procedure before delivery, but can be done to prevent defecation during labor. B is appropriate during the second stage of labor when the client's contraction is at it's peak and dilation and effacement are at maximum to help client accomplish the task of giving birth. SITUATION : [NBLUE170] Baby boy perez was delivered spontaneously following a term pregnancy. Apgar scores are 8 and 9 respectively. Routine procedures are carried out. 89. When is the APGAR Score taken? [1] A. Immediately after birth and at 30 minutes after birth B. At 5 minutes after birth and at 30 minutes after birth C. At 1 minute after birth and at 5 minutes after birth D. Immediately after birth and at 5 minutes after birth * APGAR score taken 1 minute after birth determines the initial status of the newborn while the 5 minute assessment after birth determines how well the newborn is adjusting to the extrauterine life. 90. The best way to position a newborn during the first week of life is to lay him [3] A. Prone with head slightly elevated B. On his back, flat C. On his side with his head flat on bed D. On his back with head slightly elevated * Sudden infant death syndrome occurs when the fetus is in prone position. Knowing this will allow you to eliminate A first. During the first week of life, The fetus has an immature cardiac sphincter and musculature for swallowing, Knowing this will let you eliminate B and D. Side lying position is the best position for a neonate during the first few weeks of life. This will decrease the risk of aspiration of secretion. 91. Baby boy perez has a large sebaceous glands on his nose, chin, and forehead. These are known as [1]

A. Milia B. Lanugo C. Hemangiomas D. Mongolian spots * Newborn sebaceous glands are sometimes unopened or plugged. They are called MILIA. They will disappear once the gland opens at around 2 weeks after delivery. They are characterized by a pinpoint white papule. Lanugo is the fine hair that covers the newborn. It disappears starting 2 weeks after birth. A premature infant has more lanugo than a post mature infant. Hemangiomas are vascular tumors of the skin. Mongolian spots are patches that are gray in color and are often found in sacrum or buttocks. They disappear as the child grows older. 92. Baby boy perez must be carefully observed for the first 24 hours for [2] A. Respiratory distress B. Duration of cry C. Frequency of voiding D. Range in body temperature * Range in body temperature needs to be observed and carefully monitored for the first 24 hours after delivery. A newborn has an inadequate and immature temperature regulating mechanism. RDS is observed immediately after delivery, not in a continuous 24 hour observation. Once the fetus establish a normal breathing pattern it is not anymore of a concern. RDS occurs when the Surfactants are absent or insufficient. The adequacy of these surfactants is measured by the L:S ratio [ Lecithin : Spingomyelin ] An L:S ratio of 2:1 is considered, mature and adequate to sustain fetal lung expansion and ventilation. Therefore, A child born without RDS is unlikely to have RDS in 24 hours. Another thing that is carefully observed during the first 24 hours is the meconium. Absent of meconium during the first 24 hours after birth warrants further investigation by the attending physician. 93. According to the WHO , when should the mother starts breastfeeding the infant? [4] A. Within 30 minutes after birth B. Within 12 hours after birth C. Within a day after birth D. After infant's condition stabilizes * According to the world health organization, The mother should start breastfeeding her infant within 30 minutes after birth. 94. What is the BEST and most accurate method of measuring the medication dosage for infants and children? [3]

A. Weight B. Height C. Nomogram D. Weight and Height * A nomogram is the most accurate method for measuring medication dosage for infants and children. It estimates the body surface area by drawing a line in the first column [ child's height ] towards the third column [ child's weight ]. The point in which it crosses the middle column [ BSA ] is the child's surface area. 95. The first postpartum visit should be done by the mother within [4] A. 24 hours B. 3 days C. a week D. a month * Mother should visit the health facility 4 weeks to 6 weeks after delivery. The first post partum visit by the birth attendant is done within 24 hours after delivery, the next visit will be at 1 week after delivery and the third visit is done 2 to 4 weeks after delivery. 96. The major cause of maternal mortality in the philippines is [3] A. Infection B. Hemorrhage C. Hypertension D. Other complications related to labor,delivery and puerperium * Refer to the latest survey of FHSIS in the DOH website. 97. According to the WHO, what should be the composition of a commercialized Oral rehydration salt solution? [4] A. Potassium 20 g. A. Potassium : A. Potassium : A. Potassium : : 1.5 g. ; Sodium Bicarbonate 2.5g ; Sodium Chloride 3.5g; Glucose 1.5 g. ; Sodium Bicarbonate 2.5g ; Sodium Chloride 3.5g; Glucose 10 g. 2.5 g. ; Sodium Bicarbonate 3.5g ; Sodium Chloride 4.5g; Glucose 20 g. 2.5 g. ; Sodium Bicarbonate 3.5g ; Sodium Chloride 4.5g; Glucose 10 g.

* This is the WHO ORESOL formula for the commercialized ORS. Remember PA BCG Which stands for POTASSIUM [ Pa ] SODIUM BICARBONATE [ B ] SODIUM CHLORIDE [ C ] GLUCOSE [ G ]. The numbers are easy to remember because they are just increased by 1.0 g increment starting from 1.5. Glucose however is at 20 g. So the MNEMONIC is PA BCG 1.5 2.5 3.5 20. This is the mnemonic I use and it is easy to remember that way. It is original by the way.

98. In preparing ORESOL at home, The correct composition recommnded by the DOH is [4] A. 1 glass of water, 1 pinch of salt and 2 tsp of sugar B. 1 glass of water, 2 pinch of salt and 2 tsp of sugar C. 1 glass of water, 3 pinch of salt and 4 tsp of sugar D. 1 glass of water, 1 pinch of salt and 1 tsp of sugar

99. Milk code is a law that prohibits milk commercialization or artificial feeding for up to 2 years. Which law provides its legal basis? [4] A. Senate bill 1044 B. RA 7600 C. Presidential Proclamation 147 D. EO 51 * Executive order # 51 prohibits milk commercialization or artificial feeding up to 2 years. That is why the milk commercials in the country has " BREAST MILK IS STILL BEST FOR BABIES UP TO 2 YEARS " After their presentation in accordance with EO 51. RA 7600 is the ROOMING IN / BREAST FEEDING ACT which requires the heatlh professionals to bring the baby to the mother for breastfeeding as early as possible. Senate bill # 1044 was created to implement RA 7600. Presidential Proclamation # 147 made WEDNESDAY as the national immunization day.

100. A 40 year old mother in her third trimester should avoid [4] A. Traveling B. Climbing C. Smoking D. Exercising * Mother's are not prohibited to travel, climb or exercise. If long travels are expected, Mother should have a 30 minute rest period for every 2 hours of travel [ LIPPINCOTT ]. Climbing is a very vague term used by the board examiners though I assume they are referring to climbing a flight of stairs. Anyhow, SMOKING is detrimental for both mother and child no question about it and so is ALCOHOL. In thousands of questions I answered, it never fails that HANDWASHING, AVOID SMOKING, AVOID ALCOHOL are always the answer. It still depends on the question so THINK.

Situation: During an Anatomy and Physiology class, the lecturer discussed about the female reproductive system. 101. Estrogen, one of the hormones regulating cyclic activities in female reproductive system is responsible for which effect?

a. Increases the quantity and pH of cervical mucus, causing it to become thin and watery and can be stretched to a distance of 10-13 cm. b. Inhibits the production of LH c. Increases endometrial tortuosity d. All of the above * Effects of estrogen: Inhibits the production of FSH Causes hypertrophy of the myometrium Increases the quantity and pH of cervical mucus, causing it to become thin and watery and can be stretched to a distance of 10-13 cm. Effects of Progesterone Inhibits the production of LH Increases endometrial tortuosity Increased endometrial secretions Facilitates transport of the fertilized ovum through the fallopian tubes 102. Jessa, 17 years old, is bleeding between periods of less than two weeks. This condition is an abnormality in the menstrual cycle known as: a. Metrorrhagia b. Menorrhagia c. Amenorrhea d. Dysmenorrheal * Abnormalities of Menstruation 1. Amenorrhea absence of menstrual flow 2. Dysmenorrhea painful menstruation 3. Oligomenorrhea scanty menstruation 4. Menorrhagia -excessive menstrual bleeding 5. Metrorrhagia bleeding between periods of less than 2 weeks 103. One factor of having a normal delivery is the size of the pelvis. Pelvis serves as the passageway for the passenger (fetus) during childbirth. The most ideal pelvis for childbirth is: a. Android b. Anthropoid c. Platypelloid d. Gynecoid * Gynecoid is the normal female pelvis. The inlet is well rounded. This is the most ideal pelvis for childbirth. Android male pelvis. Inlet has a narrow, shallow posterior portion and pointed anterior portion. Anthropoid transverse diameter is narrow and anteroposterior (AP) diameter of this pelvis is larger than normal. Platypelloid inlet is oval while AP diameter of this pelvis is shallow.

104. An important landmark of the pelvis that determines the distance of the descent of the head is known as: a. Linea terminalis b. Sacrum c. Ischial spines d. Ischial tuberosities * Ischial spines are the point of reference in determining the station (relationship of the fetal presenting part to the ischial spines). When the fetal head is at the level of the ischial spines the station is zero. When it is 1 cm above the ischial spines it is -1 and if 1 cm below the ischial spines it is +1. 105. The permanent cessation of menstruation is: a. Amenorrhea b. Menopause c. Oligomenorrhea d. Hypomenorrhea * The keyword here is permanent cessation. Thus, menopause is the correct answer. Amenorrhea is a temporary cessation of menses. Oligomenorrhea is a menstruation with scanty blood flow. Hypomenorrhea is an abnormally short duration of menstruation. Situation: Mrs. Donna, pregnant for 16 weeks age of gestation (AOG), visits the health care facility for her prenatal check-up with her only son, Mark. During assessment the client told the nurse that previously she got pregnant twice. The first was with her only child, Mark, who was delivered at 35 weeks AOG and the other pregnancy was terminated at about 20 weeks AOG. 106. Based on the data obtained, Mrs. Donnas GTPAL score is: a. 20111 b. 21111 c. 30111 d. 31111 * Gravida (G) number of pregnancy Term (T) number of full-term infants born (born at 37 weeks or after) Para (P) number of preterm infants born (born before 37 weeks) Abortion (A) number of spontaneous or induced abortions (pregnancy terminated before the age of viability). Age of viability is 24 weeks. Living children (L) number of living children. (Source: Maternal and Child Health Nursing by Adelle Pillitteri, 5th Ed. P.252) Since Mrs Donna has two previous pregnancies and is presently pregnant (16 weeks), G is 3. Mark, her only child was born at 35 weeks AOG which falls under the preterm category. Thus, T is zero and P is 1. The other pregnancy was terminated at 20 weeks AOG which falls under abortion, hence A=1. Mark is her only living child, thereby, L=1. Her GTPAL score is: 30111, G=3 T=0 P=1 A=1 L=1

107. Discomforts during pregnancy are discussed by the nurse to the Mrs. Donna. Which of the following, when complained by the client would alert the nurse? a. Easy fatigability b. Nausea and vomiting c. Edema of the lower extremities d. Heartburn *Morning sickness characterized by nausea and vomiting is only noted during the FIRST TRIMESTER of pregnancy (first 3 months). Excessive nausea and vomiting which persists more than 3 months is a condition called Hyperemesis gravidarum that requires immediate intervention to prevent starvation and dehydration. Management for hyperemesis gravidarum includes the administration of D5NSS 3L in 24 hours and complete bed rest. Easy fatigability is a consequence of the physiologic anemia of pregnancy (physiologic meaning it is normally expected during pregnancy, thus A is incorrect). Edema of the upper extremities not the lower extremities should alert the nurse because of the possibility of toxemia, hence C is incorrect. Heartburn during pregnancy is due to the increase progesterone which decreases gastric motility causing a reversed peristaltic wave leading to regurgitation of the stomach contents through the cardiac sphincter into the esophagus, causing irritation 108. Psychological and emotional responses of pregnant women differ. However, general emotional response has been noted during pregnancy based on their gestational age. Mrs. Donna will most likely have which emotional response towards her pregnancy? a. Presents denial disbelief and sometimes repression. b. Has personal identification of the baby and realistic plans for future of the child. c. Fantasizes the appearance of the baby. d. Verbalizes fear of death during childbirth.
*The client is in her second trimester of pregnancy (16 weeks AOG or 4 months), thus, she perceived the baby as a separate entity. Presenting denial and disbelief and sometime repression is the psychological/emotional response of a pregnant woman on her first trimester. Identifying the fetus and setting realistic plans for the childs future is noted during the third trimester of pregnancy. It is during this time also that the woman verbalizes fear of death.e

109. The nurse assisted Mrs. Donna to a dorsal recumbent position and is about to assess the fetal heart rate (FHR). Which of the following apparatus should the nurse use in auscultating for the FHR? a. Doppler apparatus b. Fetoscope c. Ultrasound d. Stethoscope
* Mrs. Donnas gestational age is 16 weeks (4 months). During this time, the fetal heart rate is audible with a Doppler apparatus. A fetal heart beat can be detected with a Doppler apparatus starting at 12 weeks AOG. By 8 weeks AOG, fetal heartbeat can be detected with an ultrasound. A fetal heart beat is

detectable with fetoscope by the 20th week AOG. (Source: Foundations of Maternal-Newborn Nursing by Murray and McKinney/Saunders 4th Ed.)

110. Mrs. Donna asked the nurse, when a fetal heart starts beating. The nurse correctly responded by stating: a. 3 weeks AOG b. 8 weeks AOG c. 12 weeks AOG d. 20 weeks AOG * Fetal heart starts beating at 3 weeks AOG. The heart at this time is consisting of two parallel tubes. By 8 weeks AOG, fetal heartbeat can be detected with an ultrasound. During 12 weeks AOG, the fetal heart rate is audible with a Doppler apparatus. A fetal heart beat is detectable with fetoscope by the 20th week AOG. (Source: Foundations of Maternal-Newborn Nursing by Murray and McKinney/Saunders 4th Ed.) Situation: Mrs. Dela Cruz is in labor and is brought to the emergency room with a ruptured bag of water. 111. The nurses initial action once the bag of water has ruptured is: a. Take the fetal heart tones b. Put the client to the bed immediately c. Perform an IE d. Take the womans temperature * The keyword is INITIAL ACTION. The important consideration before answering the question is to take a look at the situation. SITUATION: THE WOMAN IS IN THE Emergency Room or is seeking admission. A woman in labor seeking admission to the hospital (in the ER) and saying that her BOW has ruptured should BE PUT TO BED IMMEDIATELY and the fetal heart tones taken consequently. If a woman in the Labor Room says that her membranes have ruptured, the initial nursing action is to take the fetal heart tone. 112. Mrs. Dela Cruzs has contractions growing stronger which lasts for 40-60 seconds and occur approximately every 3-5 minutes. The doctor is about to perform an IE, the nurse expects that the clients cervical dilatation will be: a. 0-3 cm b. 4-7 cm c. 8-10 cm d. 11-13 cm * The nurse would expect that the clients cervical dilatation is 4-7 cm as the contraction duration and interval is noted for clients who are in the active phase of the first stage of labor. The maximum cervical dilatation is 10 cm, thus, letter D should be eliminated first. The first stage of labor (stage of dilatation) is divided into three phases. Latent phase 0-3 cm cervical dilatation; contractions are short and mild lasting 20-40

seconds and occurring approximately every 5-10 minutes. Active phase 4-7 cm cervical dilatation; contractions grow stronger, lasting 40-60 seconds and occur at approximately every 3-5 minutes. Transition phase 8-10 cm cervical dilatation; contractions reach their peak of intensity, occurring every 2-3 minutes with a duration of 60-90 seconds. 113. The doctor informed the woman that she is on station -1. Mrs. Dela Cruz asked the nurse, what does a station -1 means, the most appropriate response of the nurse is: a. It means that engagement has already occurred. b. The presenting part of your baby is at the entrance of the true pelvis or the largest diameter of the presenting part into the true pelvis. c. Your baby is still floating or ballotable d. The presenting part of your baby is at the vulvar ring of your reproductive organ. *. Station -1 means that the fetal presenting part is above the level of the ischial spines. Letter A is wrong because engagement is described as Station 0. Letter B is incorrect because the statement of nurse is describing the occurrence of engagement that is again station 0. Prior to engagement the fetus is said to be "floating" or ballottable, thus letter C is the best option. Letter D, is describing crowning which is described as Station +3 or +4. 114. The history of Mrs. Dela Cruz revealed that she is a multipara. When should the nurse transport the client from the labor room to the delivery room? a. When the cervical dilatation is 8 cm. b. When the cervical dilatation is 10 cm. c. When the cervical dilatation is 9 cm. d. When the client feels the urge to push. * Multiparas are transported to the DR when the cervical dilatation is 7-8 cm because in multiparas dilatation may proceed before effacement is completed. Effacement must occur at the end of dilatation, however, before the fetus can be safely pushed through the cervical canal; otherwise, cervical tearing could result. Primiparas are transported to the DR when the cervical dilatation is 9-10 cm. 115. Monitoring contractions is very important during labor. To monitor uterine contractions, what should the nurse do? a. Observe for the clients facial expression to know that the contraction has started or stopped. b. Instruct the client take note of the duration of her contractions. c. Offer ice chips to the woman. d. Spread the fingers lightly over the fundus to monitor the contraction. ** The nurse should spread his/her fingers lightly over the fundus to monitor the uterine contractions 116. Uterine contractions can occur because of the interplay of the contractile enzyme adenosine triphosphate and the influence some hormones. Which of the following least likely contributes to the occurrence of uterine contractions?

a. Oxytocin b. Estrogen c. Prolactin d. All of the above. *Prolactin is the hormone that produces milk in mammary glands. Uterine contractions can occur because of the interplay of the contractile enzyme adenosine triphosphate and the influence some hormones and major electrolytes which are the following: Calcium Sodium Potassium Specific contractile proteins (actin and myosin) Epinephrine and norepinephrine Oxytocin Estrogen and progesterone Prostaglandins 117. Dysfunctional labor may be caused by which of the following? a. Excessive or too early analgesia administration b. Exhausted mother c. Overdistention of the uterus d. All of the above *Dysfunctional Labor is caused by the ff: Inappropriate use of analgesia Pelvic bone contraction that has narrowed the pelvic diameter so that a client cant pass (e.g. in a client with rickets) Poor fetal position Extension rather then extension of the fetal head Overdistention of the uterus Cervical rigidity Presence of a full rectum or bladder Mother becoming exhausted from labor Primigravid status 118. The clients uterine contractions are hypotonic. The nurses top priority with hypotonic contractions during the intrapartal period is: a. Pain relief b. Psychological support c. Monitoring the lochia for possible bleeding d. Infection control *When the contractions are hypotonic, the length of labor is increased. When the cervix is dilated for a long period of time, both the uterus and fetus are at greater risk of infection. Hypotonic contractions are not exceedingly painful because of their lack of intensity. Monitoring

of bleeding through evaluation of lochia is done during the postpartum period not the intrapartum period. 119. For a woman experiencing hypotonic contractions, what should be done initially? a. Obtain an ultrasonic result b. Infusion of oxytocin c. Administration of analgesia d. Amniotomy * Initially, the nurse should obtain an ultrasonic confirmation ruling out a CPD or cephalopelvic disproportion. Thus, A is the best answer. Oxytocin is infused after the CPD is ruled out, because if CPD is present CS will be done. Analgesic administration will further decrease the intensity of uterine contractions as its inappropriate use is one of the reasons why hypotonic contractions occur. Amniotomy (artificial rupture of membrane) may be done after oxytocin is infused to speed up the labor 120. The most important nursing consideration in a postpartal woman with a hypotonic contraction is: a. Assessment for infection b. Assessment for bleeding c. Assessment for FHR d. Assessment for womans coping mechanism * During the postpartum period, the uterus should be palpated and lochia should be assessed because contractions after birth may also be hypotonic that will result to bleeding. Situation: Bleeding during pregnancy is a serious case and should be managed immediately. 121. Mrs. Diane is diagnosed with Placenta Previa. The main difference with the bleeding in placenta previa and abruption placenta is that placenta previa has: a. Painful bleeding b. Rigid abdomen c. Bright-red blood d. Blood filled with clots * In placenta previa the bleeding that occurs is abrupt, painless, bright-red and sudden to frighten a woman. With abruption placenta, the bleeding is painful, the abdomen is rigid or board-like and the blood is dark-red or filled with clots. 122. In caring for a client diagnosed with placenta previa, the nurse should avoid which of the following? a. Inspecting the perineum b. Performing a Kleihauer-Betke test c. Performing a pelvic examination d. All of the above

* Never attempt a pelvic or rectal examination with painless bleeding late in pregnancy because any agitation of the cervix when there is placenta previa may initiate massive hemorrhage, possibly fatal to both the mother and the fetus. The perineum should be assessed or observed or inspected for bleeding by looking over the

123. For the nurse to distinguish that the bleeding of the patient is placenta previa or abruption placenta what should she ask the woman? a. Whether there was accompanying pain b. What she has done for bleeding c. Estimation of blood loss d. All of the above * placenta previa presents bleeding without pain whilst the bleeding in abruptio placenta is painful. 124. Continued bleeding may result to fetal distress. The nurse knows that the fetus is being compromised when she observed or note which of the following: a. Fetal tachycardia b. Fetal bradycardia c. Fetal thrashing d. All of the above * Signs of fetal distress include: tachycardia, bradycardia, fetal thrashing and meconium-stained amniotic fluid. 125. A woman in labor is diagnosed with abruption placenta. The nurse would expect which findings in the clients history that may contribute to the occurrence of the complication? a. Age of 24 years old b. Cigarette smoking c. Sleeping 8 hours per night d. Sitting for long period * Predisposing factors for abruptio placenta: Advanced maternal age Short-umbilical cord Chronic hypertensive disease PIH Direct trauma Vasoconstriction from cocaine or cigarette use
126. Accompanied by her husband, a patient seeks admission to the labor and delivery area. The client states that she is in labor, and says she attended the hospital clinic for prenatal care. Which question should the nurse ask her first?

a. b. c. d.

Do you have any chronic illness? Do you have any allergies? What is your expected due date? Who will be with you during labor?

* When obtaining the history of a patient who may be in labor, the nurses highest priority is to determine her current status, particularly her due date, gravidity, and parity. Gravidity and parity affect the duration of labor and the potential for labor complications. Later, the nurse should ask about chronic illness, allergies, and support persons.

127. A patient is in the second stage of labor. During this stage, how frequently should the nurse in charge assess her uterine contractions? a. Every 5 minutes b. Every 15 minutes c. Every 30 minutes d. Every 60 minutes *. During the second stage of labor, the nurse should assess the strength, frequency, and duration of contraction every 15 minutes. If maternal or fetal problems are detected, more frequent monitoring is necessary. An interval of 30 to 60 minutes between assessments is too long because of variations in the length and duration of patients labor. 128. A patient is in last trimester of pregnancy. Nurse Jane should instruct her to notify her primary health care provider immediately if she notices: a. Blurred vision b. Hemorrhoids c. Increased vaginal mucus d. Shortness of breath on exertion * Blurred vision of other visual disturbance, excessive weight gain, edema, and increased blood pressure may signal severe preeclampsia. This condition may lead to eclampsia, which has potentially serious consequences for both the patient and fetus. Although hemorrhoids may be a problem during pregnancy, they do not require immediate attention. Increased vaginal mucus and dyspnea on exertion are expected as pregnancy progresses. 129. The nurse in charge is reviewing a patients prenatal history. Which finding indicates a genetic risk factor? a. The patient is 25 years old b. The patient has a child with cystic fibrosis c. The patient was exposed to rubella at 36 weeks gestation d. The patient has a history of preterm labor at 32 weeks gestation * Cystic fibrosis is a recessive trait; each offspring has a one in four chance of having the trait or the disorder. Maternal age is not a risk factor until age 35, when the incidence of

chromosomal defects increases. Maternal exposure to rubella during the first trimester may cause congenital defects. Although a history or preterm labor may place the patient at risk for preterm labor, it does not correlate with genetic defects. 130. A adult female patient is using the rhythm (calendar-basal body temperature) method of family planning. In this method, the unsafe period for sexual intercourse is indicated by; a. Return preovulatory basal body temperature b. Basal body temperature increase of 0.1 degrees to 0.2 degrees on the 2nd or 3rd day of cycle c. 3 full days of elevated basal body temperature and clear, thin cervical mucus d. Breast tenderness and mittelschmerz * Ovulation (the period when pregnancy can occur) is accompanied by a basal body temperature increase of 0.7 degrees F to 0.8 degrees F and clear, thin cervical mucus. A return to the preovulatory body temperature indicates a safe period for sexual intercourse. A slight rise in basal temperature early in the cycle is not significant. Breast tenderness and mittelschmerz are not reliable indicators of ovulation. 131. During a nonstress test (NST), the electronic tracing displays a relatively flat line for fetal movement, making it difficult to evaluate the fetal heart rate (FHR). To mark the strip, the nurse in charge should instruct the client to push the control button at which time? a. At the beginning of each fetal movement b. At the beginning of each contraction c. After every three fetal movements d. At the end of fetal movement * An NST assesses the FHR during fetal movement. In a healthy fetus, the FHR accelerates with each movement. By pushing the control button when a fetal movement starts, the client marks the strip to allow easy correlation of fetal movement with the FHR. The FHR is assessed during uterine contractions in the oxytocin contraction test, not the NST. Pushing the control button after every three fetal movements or at the end of fetal movement wouldnt allow accurate comparison of fetal movement and FHR changes. 132. When evaluating a clients knowledge of symptoms to report during her pregnancy, which statement would indicate to the nurse in charge that the client understands the information given to her? a. Ill report increased frequency of urination. b. If I have blurred or double vision, I should call the clinic immediately. c. If I feel tired after resting, I should report it immediately. d. Nausea should be reported immediately. * Blurred or double vision may indicate hypertension or preeclampsia and should be reported immediately. Urinary frequency is a common problem during pregnancy caused by increased weight pressure on the bladder from the uterus. Clients generally experience fatigue and nausea during pregnancy. 133. When assessing a client during her first prenatal visit, the nurse discovers that the client had a reduction mammoplasty. The mother indicates she wants to breast-feed. What

information should the nurse give to this mother regarding breast-feeding success? a. Its contraindicated for you to breast-feed following this type of surgery. b. I support your commitment; however, you may have to supplement each feeding with formula. c. You should check with your surgeon to determine whether breast-feeding would be possible. d. You should be able to breast-feed without difficulty. * Recent breast reduction surgeries are done in a way to protect the milk sacs and ducts, so breast-feeding after surgery is possible. Still, its good to check with the surgeon to determine what breast reduction procedure was done. There is the possibility that reduction surgery may have decreased the mothers ability to meet all of her babys nutritional needs, and some supplemental feeding may be required. Preparing the mother for this possibility is extremely important because the clients psychological adaptation to mothering may be dependent on how successfully she breast-feeds. 134. Following a precipitous delivery, examination of the clients vagina reveals a fourthdegree laceration. Which of the following would be contraindicated when caring for this client? a. Applying cold to limit edema during the first 12 to 24 hours b. Instructing the client to use two or more peripads to cushion the area c. Instructing the client on the use of sitz baths if ordered d. Instructing the client about the importance of perineal (Kegel) exercises * Using two or more peripads would do little to reduce the pain or promote perineal healing. Cold applications, sitz baths, and Kegel exercises are important measures when the client has a fourth-degree laceration. 135. A client makes a routine visit to the prenatal clinic. Although shes 14 weeks pregnant, the size of her uterus approximates that in an 18- to 20-week pregnancy. Dr. Diaz diagnoses gestational trophoblastic disease and orders ultrasonography. The nurse expects ultrasonography to reveal: a. an empty gestational sac. b. grapelike clusters. c. a severely malformed fetus. d. an extrauterine pregnancy. * In a client with gestational trophoblastic disease, an ultrasound performed after the 3rd month shows grapelike clusters of transparent vesicles rather than a fetus. The vesicles contain a clear fluid and may involve all or part of the decidual lining of the uterus. Usually no embryo (and therefore no fetus) is present because it has been absorbed. Because there is no fetus, there can be no extrauterine pregnancy. An extrauterine pregnancy is seen with an ectopic pregnancy. 136. After completing a second vaginal examination of a client in labor, the nurse-midwife determines that the fetus is in the right occiput anterior position and at 1 station. Based on these findings, the nurse-midwife knows that the fetal presenting part is:

a. b. c. d.

1 cm below the ischial spines. directly in line with the ischial spines. 1 cm above the ischial spines. in no relationship to the ischial spines.

* Fetal station the relationship of the fetal presenting part to the maternal ischial spines is described in the number of centimeters above or below the spines. A presenting part above the ischial spines is designated as 1, 2, or 3. A presenting part below the ischial spines, as +1, +2, or +3.

137. Which of the following would be inappropriate to assess in a mother whos breastfeeding? a. The attachment of the baby to the breast. b. The mothers comfort level with positioning the baby. c. Audible swallowing. d. The babys lips smacking * Assessing the attachment process for breast-feeding should include all of the answers except the smacking of lips. A baby whos smacking his lips isnt well attached and can injure the mothers nipples. 138. During a prenatal visit at 4 months gestation, a pregnant client asks whether tests can be done to identify fetal abnormalities. Between 18 and 40 weeks gestation, which procedure is used to detect fetal anomalies? a. Amniocentesis. b. Chorionic villi sampling. c. Fetoscopy. d. Ultrasound * Ultrasound is used between 18 and 40 weeks gestation to identify normal fetal growth and detect fetal anomalies and other problems. Amniocentesis is done during the third trimester to determine fetal lung maturity. Chorionic villi sampling is performed at 8 to 12 weeks gestation to detect genetic disease. Fetoscopy is done at approximately 18 weeks gestation to observe the fetus directly and obtain a skin or blood sample. 139. A client, 30 weeks pregnant, is scheduled for a biophysical profile (BPP) to evaluate the health of her fetus. Her BPP score is 8. What does this score indicate? a. b. c. d. The fetus should be delivered within 24 hours. The client should repeat the test in 24 hours. The fetus isnt in distress at this time. The client should repeat the test in 1 week.

* The BPP evaluates fetal health by assessing five variables: fetal breathing movements, gross body movements, fetal tone, reactive fetal heart rate, and qualitative amniotic fluid volume. A normal response for each variable receives 2 points; an abnormal response

receives 0 points. A score between 8 and 10 is considered normal, indicating that the fetus has a low risk of oxygen deprivation and isnt in distress. A fetus with a score of 6 or lower is at risk for asphyxia and premature birth; this score warrants detailed investigation. The BPP may or may not be repeated if the score isnt within normal limits. 140. A client whos 36 weeks pregnant comes to the clinic for a prenatal checkup. To assess the clients preparation for parenting, the nurse might ask which question? a. Are you planning to have epidural anesthesia? b. Have you begun prenatal classes? c. What changes have you made at home to get ready for the baby? d. Can you tell me about the meals you typically eat each day? * During the third trimester, the pregnant client typically perceives the fetus as a separate being. To verify that this has occurred, the nurse should ask whether she has made appropriate changes at home such as obtaining infant supplies and equipment. The type of anesthesia planned doesnt reflect the clients preparation for parenting. The client should have begun prenatal classes earlier in the pregnancy. The nurse should have obtained dietary information during the first trimester to give the client time to make any necessary changes. 141. A client whos admitted to labor and delivery has the following assessment findings: gravida 2 para 1, estimated 40 weeks gestation, contractions 2 minutes apart, lasting 45 seconds, vertex +4 station. Which of the following would be the priority at this time? a. Placing the client in bed to begin fetal monitoring. b. Preparing for immediate delivery. c. Checking for ruptured membranes. d. Providing comfort measures. * This question requires an understanding of station as part of the intrapartal assessment process. Based on the clients assessment findings, this client is ready for delivery, which is the nurses top priority. Placing the client in bed, checking for ruptured membranes, and providing comfort measures could be done, but the priority here is immediate delivery.

142. Nurse Roy is caring for a client in labor. The external fetal monitor shows a pattern of variable decelerations in fetal heart rate. What should the nurse do first? a. Change the clients position. b. Prepare for emergency cesarean section. c. Check for placenta previa. d. Administer oxygen. * Variable decelerations in fetal heart rate are an ominous sign, indicating compression of the umbilical cord. Changing the clients position from supine to side-lying may immediately correct the problem. An emergency cesarean section is necessary only if other measures, such as changing position and amnioinfusion with sterile saline, prove unsuccessful. Administering oxygen may be helpful, but the priority is to change the womans position and relieve cord compression. 143. The nurse in charge is caring for a postpartum client who had a vaginal delivery with a

midline episiotomy. Which nursing diagnosis takes priority for this client? a. Risk for deficient fluid volume related to hemorrhage b. Risk for infection related to the type of delivery c. Pain related to the type of incision d. Urinary retention related to periurethral edema * Hemorrhage jeopardizes the clients oxygen supply the first priority among human physiologic needs. Therefore, the nursing diagnosis of Risk for deficient fluid volume related to hemorrhage takes priority over diagnoses of Risk for infection, Pain, and Urinary retention 144. Which change would the nurse identify as a progressive physiological change in postpartum period? a. Lactation b. Lochia c. Uterine involution d. Diuresis * Lactation is an example of a progressive physiological change that occurs during the postpartum period. 145. A 39-year-old at 37 weeks gestation is admitted to the hospital with complaints of vaginal bleeding following the use of cocaine 1 hour earlier. Which complication is most likely causing the clients complaint of vaginal bleeding? a. Placenta previa b. Abruptio placenta c. Ectopic pregnancy d. Spontaneous abortion * The major maternal adverse reactions from cocaine use in pregnancy include spontaneous abortion first, not third, trimester abortion and abruption placentae. 146. A client with type 1 diabetes mellitus whos a multigravida visits the clinic at 27 weeks gestation. The nurse should instruct the client that for most pregnant women with type 1 diabetes mellitus: a. Weekly fetal movement counts are made by the mother. b. Contraction stress testing is performed weekly. c. Induction of labor is begun at 34 weeks gestation. d. Nonstress testing is performed weekly until 32 weeks gestation * For most clients with type 1 diabetes mellitus, nonstress testing is done weekly until 32 weeks gestation and twice a week to assess fetal well-being. 147. When administering magnesium sulfate to a client with preeclampsia, the nurse understands that this drug is given to:

a. Prevent seizures b. Reduce blood pressure c. Slow the process of labor d. Increase dieresis * The chemical makeup of magnesium is similar to that of calcium and, therefore, magnesium will act like calcium in the body. As a result, magnesium will block seizure activity in a hyper stimulated neurologic system by interfering with signal transmission at the neuromascular junction. 148. Whats the approximate time that the blastocyst spends traveling to the uterus for implantation? a. 2 days b. 7 days c. 10 days d. 14 weeks * The blastocyst takes approximately 1 week to travel to the uterus for implantation. 149. After teaching a pregnant woman who is in labor about the purpose of the episiotomy, which of the following purposes stated by the client would indicate to the nurse that the teaching was effective? a. Shortens the second stage of labor b. Enlarges the pelvic inlet c. Prevents perineal edema d. Ensures quick placenta delivery * An episiotomy serves several purposes. It shortens the second stage of labor, substitutes a clean surgical incision for a tear, and decreases undue stretching of perineal muscles. An episiotomy helps prevent tearing of the rectum but it does not necessarily relieves pressure on the rectum. Tearing may still occur. 150. A primigravida client at about 35 weeks gestation in active labor has had no prenatal care and admits to cocaine use during the pregnancy. Which of the following persons must the nurse notify? a. Nursing unit manager so appropriate agencies can be notified b. Head of the hospitals security department c. Chaplain in case the fetus dies in utero d. Physician who will attend the delivery of the infant * The fetus of a cocaine-addicted mother is at risk for hypoxia, meconium aspiration, and intrauterine growth retardation (IUGR). Therefore, the nurse must notify the physician of the clients cocaine use because this knowledge will influence the care of the client and neonate. The information is used only in relation to the clients care. 151. When preparing a teaching plan for a client who is to receive a rubella vaccine during the postpartum period, the nurse in charge should include which of the following?

a. The vaccine prevents a future fetus from developing congenital anomalies b. Pregnancy should be avoided for 3 months after the immunization c. The client should avoid contact with children diagnosed with rubella d. The injection will provide immunity against the 7-day measles. * After administration of rubella vaccine, the client should be instructed to avoid pregnancy for at least 3 months to prevent the possibility of the vaccines toxic effects to the fetus. 152. A client with eclampsia begins to experience a seizure. Which of the following would the nurse in charge do first? a. Pad the side rails b. Place a pillow under the left buttock c. Insert a padded tongue blade into the mouth d. Maintain a patent airway * The priority for the pregnant client having a seizure is to maintain a patent airway to ensure adequate oxygenation to the mother and the fetus. Additionally, oxygen may be administered by face mask to prevent fetal hypoxia. 153. While caring for a multigravida client in early labor in a birthing center, which of the following foods would be best if the client requests a snack? a. Yogurt b. Cereal with milk c. Vegetable soup d. Peanut butter cookies * In some birth settings, intravenous therapy is not used with low-risk clients. Thus, clients in early labor are encouraged to eat healthy snacks and drink fluid to avoid dehydration. Yogurt, which is an excellent source of calcium and riboflavin, is soft and easily digested. During pregnancy, gastric emptying time is delayed. In most hospital settings, clients are allowed only ice chips or clear liquids. 154. The multigravida mother with a history of rapid labor who us in active labor calls out to the nurse, The baby is coming! which of the following would be the nurses first action? a. Inspect the perineum b. Time the contractions c. Auscultate the fetal heart rate d. Contact the birth attendant * When the client says the baby is coming, the nurse should first inspect the perineum and observe for crowning to validate the clients statement. If the client is not delivering precipitously, the nurse can calm her and use appropriate breathing techniques. 155. While assessing a primipara during the immediate postpartum period, the nurse in charge plans to use both hands to assess the clients fundus to:

a. Prevent uterine inversion b. Promote uterine involution c. Hasten the puerperium period d. Determine the size of the fundus

* Using both hands to assess the fundus is useful for the prevention of uterine inversion.

156. A patient on labor is offered ice chips. What is the purpose of giving ice chips to a woman on labor?

a.To reduce a womans anxiety b.To prevent impulses reach the distracted brain c.To promote cutaneous stimulation d.To allow more blood to pool in the uterus, thus alleviating pain

* In Lamaze method one technique of alleviating pain is by cutaneous stimulation. It is done by effleurage, sacral pressure, thermal stimulation (offering ice chips) and positioning. Preventing impulses to reach the brain is employing the distraction or imagery technique. Reducing a womans anxiety is done by a continuous support system. Allowing more blood to pool in the uterus to alleviate pain is a principle in Dick-Read method.

157. The hallmark techniques of Laborer method are the following except: a. Newborn warm bath b. husband as a throughout labor and delivery c. infant massage d. Immediate maternal-newborn bonding * Infant massage, a warm bath and immediate bonding with the mother are all hallmarks of the Leboyer method. Bradley childbirth method was the first technique to include the father as a support person during delivery. 158. A pregnant woman is diagnosed with toxemia. The following are signs of toxemia apart from:

a.Edema of the lower extremities b.Sudden Weight gain c.Blurry vision d.Protein in urine
*Toxemia or Pregnancy induced hypertension has the following manifestations: Edema of the face, hands, ankle and feet, proteinuria, visual disturbances, sudden weight gain, headache, fever. Edema of lower extremity alone is not a sign of toxemia as it is a normal adaptation to pregnancy that is the result of the poor circulation in the lower extremities due to the pressure from the gravid uterus. It is managed by raising the legs above the hip level. Edema of the Lower Extremities plus the upper extremities is a sign of toxemia. 159. The obstetrician who pointed out that a baby born in a less stressful environment is more content is:

a.Dr. Grantly Dick-Read b.Dr. Robert Bradley c.Dr. Ferdinand Lamaze d.Dr. Frederick Leboyer * Leboyer childbirth method was introduced by Frederick Leboyer. He is a French obstetrician who
believed that the traditional hospital births of the time (1975) were traumatic for the infant. In 1975, he published a bookentitled Birth without Violence, where depth of a newborns sensitivity and the importance of how the baby is handled by the people around him were emphasized. He pointed out that babies born in a less stressful environment were more content. 160. Mrs. White, a pregnant woman, is 54 tall and weighs 130 lbs. Calculate her body mass index.

a.22 BMI b.30 BMI c.24 BMI d.28 BMI * 22 BMI To compute for BMI 1. Convert weight into kilograms: 130 lbs / 2.2= 59.09 kg (1 kg=2.2 lbs) 2.
Convert height into cm: 54 ( 5 feet 4 inches) 1 foot=12 inches; 1 inch = 2.54 cm CONVERT TO INCH FIRST: 5 x 12 = 60 inches + 4 inches = 64 inches INCN TO CM: 64 x 2.54 = 162.56 cm 3. Square the height in meters (1 m = 100 cm) CONVERT CM TO METER: 162.56 / 100 = 1.6256 m or 1.63 m SQUARE THE HEIGHT IN METER: 1.63 x 1.63 = 2.6569 or 2.66 4. Divide weight (kg) by meter squared 59.09 / 2.66 = 22 BMI

161. Physical assessment is very important during prenatal visits to evaluate the overall health status of a pregnant
woman. Weight is one indicator of the health status. A pregnant woman on her second trimester of pregnancy should gain how many kilograms per month?

a.0.45 kg b.1.82 kg c.1.36 kg d.2 kg

* Normal weight gain pattern in pregnancy is 1 lb (0.45 kg) per month in the first trimester and 1 lb per week in the second and third trimesters. Meaning to say, in the second and third trimester the pregnant woman should be gaining 4 lbs (1.82 kg) per month. To easily remember the pattern of weight gain in pregnancy is 3-12-12. 3 lbs in the first trimester and 12 lbs each in the second and third trimester

162. During a prenatal visit the pregnant woman is complaining of itchy, foul-smelling vaginal discharges that are
frothy and cream-colored. This is a hallmark sign of:

a.Moniliasis b.Candidiasis c.Syphilis d.Vaginitis


* Frothy, cream-colored vaginal discharges, vulvar edema and hyperemia are hallmark signs of Trichomoniasis or vaginitis. Moniliasis or Candidiasis is caused by a fungus Candida Albicans (Monilia). Symptoms include: white patchy and cheese like particles that adheres to the vaginal wall, irritatingly itchy and foul smelling vaginal discharges. Chancres on vulva and non-itchy rash covering the whole body or appearing in patches are manifestations of syphilis. 163. A pregnant woman is asking about the possible causes of morning sickness. The nurse least likely identifies the
cause by stating which of the following?

a.It is due to the increased human chorionic gonadotrophin levels during pregnancy. b.It may be due to increased acidity of the GI tract. c.It may be the result of increased progesterone level which in return decreases gastric motility thus causing reverse peristaltic waves. d.Some cases of morning sickness can also be due to emotional factors.
* Heartburn not morning sickness is the result of increased progesterone level which in return decreases gastric motility thus causing reverse peristaltic waves. 164. A sensitive and unobtrusive technique of care while respecting the natural process and promoting a peaceful
atmosphere at birth will help the baby to be born with a minimum stress. This is least likely achieved by:

a.Keeping the room noise to a minimum b.Not delaying the cutting of the cord c.Giving the newborn a warm bath after delivery d.Not assisting or pulling the babys head in expulsion
* Because of the word least likely the correct option is B. The method employed in promoting a sensitive and unobtrusive technique of care while respecting the natural process and promoting a

peaceful atmosphere at birth to help the baby be born with a minimum stress is LeBoyer Childbirth Method. To decrease the trauma at birth this technique is an advocate of the following: The birthing room is darkened or dimmed. Doing so prevents sudden contrast of light that might distress the newborn. Leboyer thought that this is less shocking for the neonates eyes that have been in semidarkness for several months. A soft music is played or at least harsh noises are kept to a minimum. All the talking inside the delivery room is done by whispering. The room is kept pleasantly warm, not chilled, to help the newborn be comfortable with the new environment and adapt to extrauterine life more easily. Newborns are handled gently and carefully. Leboyer recommends not pulling the babys head to completely allow a natural childbirth. The umbilical cord is cut late (It is cut after it has stopped pulsating). According to Leboyer, this allows the newborn to continue receiving the maternal hormones and oxygen carrying red blood cells. This is also done to allow the baby to breathe when he is ready and not have to be rushed. The infant receives a warm bath immediately after birth for relaxation. Before maternal and newborn bonding is done, the infant is massage to ease crying. Nowadays, mothers do this massaging to encourage immediate bonding. Placing the infant in the mothers abdomen is done for maternal-neonate bonding. 165. The priority management for Hyperemesis Gravidarum is:

a.Easting dry toast or crackers 30 minutes before arising in the morning b.Complete bed rest (CBR) c.D5NSS 3L in 24 hours d.Administer Milk of Magnesia
* The priority treatment for Hyperemesis gravidarum is giving 3 L of D5NSS for 24 hours. CBR is also important but the main treatment is D5NSS to replace the lost fluids. Eating dry crackers before arising in the morning is the management for morning sickness not Hyperemesis gravidarum. Milk of magnesia is an antacid used to manage heartburn in pregnant woman. 166. What is the best interpretation of Mrs. Whites calculated BMI?

a.Underweight b.Normal weight c.Overweight d.Obese


* Normal weight a. Underweight under 18.5 BMI b. Normal weight 18.5-24.9 BMI c. Overweight 25-29.9 BMI d. Obese above 30.0 167. Childbirth education is vital for a successful and comfortable childbirth method. The first method that includes
the father as a support person while embracing the belief that childbirth is a natural event and with the right preparation most women can have spontaneous and unmedicated vaginal births is:

a.Lamaze method b.Leboyer method

c.Bradley method d.Grantly Dick-Read method


* Bradley childbirth method was the first technique to include the father as a support person during delivery. This method embraces the belief that childbirth is a natural event and with the right preparation most women can have spontaneous and unmedicated vaginal births. Lamaze method is a prepared labor and delivery technique. It is often called psychoprophylactic method as it uses the mind (psyche) to prevent labor pains (prophylaxis). Leboyer childbirth method sometimes termed as birth without violence, views birth as a stressful experience for the newborn. The focus of this method is to primarily improve the quality of the birth experience for the baby. Grantly Dick-read Method is a psychophysiological preparation for childbirth. This technique alleviates pain during childbirth by blocking feelings of fear and tension. 168. The entitled Childbirth without Fear is authored by:

a.Dr. Grantly Dick-Read b.Dr. Robert Bradley c.Dr. Ferdinand Lamaze d.Dr. Frederick Leboyer
* Dr. Grantly Dick-Read is an English obstetrician who was practicing medicine in 1900s. On 1933 his book, Childbirth without Fear, was published. He explained in this publication that no physiological function in a persons body can give rise to pain under the condition that it is of normal course of health. Unless a disease process is taking place, pain or agony would associate delivery Bradley Method was named after an American obstetrician Dr. Robert Bradley. Husband Coached Childbirth, was written by Dr. Bradley and his principles and theories paved the way for fathers to be present in the delivery room. This changed the face of childbirth around the globe. Lamaze childbirth method is an alternative to the use of medical intervention during childbirth. The method was originally developed in Russia (based on Pavlovs conditioning studies) but was popularized by a French obstetrician Dr. Ferdinand Lamaze. After watching a woman gave birth in Russia Dr. Lamaze developed his own system of painless childbirth. Leboyer childbirth method was introduced by Frederick Leboyer. He is a French obstetrician who believed that the traditional hospital births of the time (1975) were traumatic for the infant. In 1975, he published a book entitled Birth without Violence, where depth of a newborns sensitivity and the importance of how the baby is handled by the people around him were emphasized. He pointed out that babies born in a less stressful environment were more content. 169. The easiest method for determining if a womans caloric intake is adequate is assessing:

a.Food intake b.Bowel pattern c.Weight she is gaining d.A typical day history or a 24 hour nutrition recall

* The easiest method for determining if a womans caloric intake is adequate is assessing the weight she is gaining. (Pillitteri, 5th Ed). A typical day history or a 24 hour nutrition recall is the best method for assessing overall nutritional intake 170. The following concepts are emphasized in a Bradley class apart from:

a.Avoidance of drugs throughout the pregnancy b.Natural breathing methods (without patterns) c.Coaching techniques d.Woman should change position during labor
* Encouraging a woman on labor to change position during labor is a concept of Lamaze method to stimulate cutaneous stimulation. The following concepts are taught in Bradley Classes: Natural birth process Active involvement of the husband as a coach Proper nutrition during pregnancy Avoidance of drugs throughout the pregnancy Natural breathing methods (without patterns) Coaching techniques Natural pain coping methods Maternal-newborn bonding Breastfeeding Education on emergent situations throughout pregnancy and delivery (CS) Postpartum and newborn care Responsible parenthood 171. During one of the sessions a pregnant mother is expressing concern on her baby. She is voicing apprehensions
on the outcome of her delivery that might be stressful to her baby. Birth without violence is under which theory?

a.Lamaze method b.Leboyer method c.Bradley method d.Grantly Dick-Read method


* Leboyer childbirth method sometimes termed as birth without violence, views birth as a stressful experience for the newborn. The focus of this method is to primarily improve the quality of the birth experience for the baby 172. The total weight gain in pregnancy for women who have multiple fetuses is:

a.20-25 lbs b.25-35 lbs c.35-40 lbs d.40-45 lbs


* For women who have twins or multiple fetuses the weight gain should be 1 lb/week for a total of 4045 lbs. (Adelle Pillitteri, 5th Ed p.302). 25-35 lbs is the average and recommended weight gain for pregnant woman with one fetus only. 173. The obstetrician who associated childbirth method with Pavlovs conditioning studies is:

a.Dr. Grantly Dick-Read b.Dr. Robert Bradley

c.Dr. Ferdinand Lamaze d.Dr. Frederick Leboyer


* Lamaze childbirth method is an alternative to the use of medical intervention during childbirth. The method was originally developed in Russia (based on Pavlovs conditioning studies) but was popularized by a French obstetrician Dr. Ferdinand Lamaze. After watching a woman gave birth in Russia Dr. Lamaze developed his own system of painless childbirth. 174. A diagnosis of Moniliasis is made. The nurse is aware that symptoms of Moniliasis typically has a symptom of:

a.White patchy and cheese like particles that adheres to the vaginal wall b.Chancres on the vulva c.Frothy, cream-colored vaginal discharge d.A non-itchy rash covering the whole body or appearing in patches
* Moniliasis or Candidiasis is caused by a fungus Candida Albicans (Monilia). Symptoms include: white patchy and cheese like particles that adheres to the vaginal wall, irritatingly itchy and foul smelling vaginal discharges. Chancres on vulva and non-itchy rash covering the whole body or appearing in patches are manifestations of syphilis. Frothy, cream-colored vaginal discharges, vulvar edema and hyperemia are hallmark signs of Trichomoniasis or vaginitis. 175. The nurse is calculating a pregnant womans BMI. Her calculated result is 26 BMI. This BMI belongs to the
category of:

a.Underweight b.Normal weight c.Overweight d.Obese


* Underweight under 18.5 BMI b. Normal weight 18.5-24.9 BMI c. Overweight 25-29.9 BMI d. Obese above 30.0 176. Myra, a 25-year old woman, on labor firmly told her doctor not to give her pain medications during labor and
delivery. She believes that it is best for her baby if she would not receive pain medications and no physiological events such as birth can give rise to pain under the condition that she is free from underlying diseases. Myra insisted that knowing what are expected and being tension-free during the delivery process would need no pain medications. Her principle is based on the philosophy of which method?

a.Lamaze method b.Leboyer method c.Bradley method d.Grantly Dick-Read method


* Grantly Dick-read Method is a psychophysiological preparation for childbirth. This technique alleviates pain during childbirth by blocking feelings of fear and tension. Dr. Dick-Read postulated that this fear felt

by women during delivery causes the blood to be carried away from the uterus to be used by the muscles needing it due to perceived pain. This results to an unoxygenated uterus which would be unable to perform its function well leading to pain, a phenomenon called the fear-tension-pain syndrome. Dick-Read believed that presence of fear and tension result to labor pains. Thus, eliminating fear would increase blood supply to the uterus and alleviating labor pains. He also identified that laboring women needed constant emotional support to help them cope. 177. The nurse is conducting a health teaching on expectant mothers. She is stressing the importance of childbirth
education classes in one of her discussions. Most pregnant women are expressing concern about their fear on the upcoming labor. The nurse is discussing about the ways of alleviating labor pains using the mind which is based on which childbirth principle?

a.Lamaze method b.Leboyer method c.Bradley method d.Grantly Dick-Read method


* Lamaze method is a prepared labor and delivery technique. It is often called psychoprophylactic method as it uses the mind (psyche) to prevent labor pains (prophylaxis). The Lamaze method of prepared childbirth is based on the gating control theory of pain relief. It involves concentration and conditioning to help the woman respond to contractions with relaxation from techniques thereby alleviating pain. This is based on the theory that through stimulus-response conditioning, women can learn to use controlled breathing to reduce the pain in the labor. Lamaze method is the most popular approach used to today. 178. The structure which contains high amount of estrogen is called:

a.Corpus albicans b.Corpus luteum c.Primordial follicles d.Graafian follicles


* Graafian follicles are structures containing high amounts of estrogen. Graafian follicle are derived from primordial follicles. Corpus Luteum is the structure containing high amounts of progesterone and is yellow in color. If no fertilization occurs corpus luteum turns white and is now called Corpus Albicans. 179.

Das könnte Ihnen auch gefallen